You are on page 1of 37

Cmo ensear tica, Capital Social y Desarrollo en la Universidad?

Mdulo 4: tica y Capital Social: contenidos temticos

Mdulo 4
Contenidos Temticas: Capital Social y Desarrollo

Temario

Definiciones del Capital Social


Tipos de Capital Social
Capital Social, economa y democracia
Qu es lo que genera Capital Social?
Redefiniendo el desarrollo desde el enfoque de Capital Social
Qu es lo que genera capital antisocial?
Ensear el Capital Social de modo ldico
La contradiccin: modernizacin y mal desarrollo
Los retos del desarrollo desde el enfoque de Etica, Capital Social y Desarrollo
Conclusiones
Preguntas para el Foro
Actividades obligatorias

Definiciones del Capital Social


La fuerza del cocodrilo es el agua.
Proverbio Bant
La nocin de Capital Social ha provocado muchos debates en las ciencias sociales desde su
introduccin en la dcada de los 80, y a pesar de ser un concepto cada vez ms importante
para la formulacin de las polticas sociales del continente, posee muy poco consenso entre los
investigadores. Sin duda, tiene que ver con la complejidad que resalta de las investigaciones y
el paradigma peculiar de sociedad al cual se refiere: la nocin de Capital Social nos invita a
comprender la sociedad como un todo de relaciones, en el cual el lazo social lo constituye la
intersubjetividad bajo forma de redes de reciprocidad y confianza.
La sociedad, como conjunto de "conductas plurales" (Weber), situaciones sociales compartidas
que generan identificacin y redes de cooperacin dentro de una clase social (Marx), se
mantiene cohesionada gracias a "conciencias colectivas" (Durkheim), sistemas simblicos,
valores compartidos y las solidaridades que las acompaan.
Con esta visin de la sociedad, estamos lejos del Leviatn de Hobbes
http://www.cibernous.com/autores/hobbes/teoria/biografia.html y de la competencia permanente
entre individuos atomizados, egostas y calculadores que la economa neoclsica quiere
hacernos asumir, confundiendo una definicin de Mercado hiperliberal con un modelo de
Sociedad. El enfoque de Capital Social nos conduce a pensar que la "mano invisible" que opera
el milagro del funcionamiento del sistema entero, no es otra cosa que las redes de relaciones de
reciprocidad y vnculo entre las personas. Sin embargo, los inventores de la nocin de Capital
Social que son James Coleman y Pierre Bourdieu (http://www.iigov.org/documentos/?p=3_0106
1

Portal Educativo de las Amricas


http://www.educoea.org

Cmo ensear tica, Capital Social y Desarrollo en la Universidad?


Mdulo 4: tica y Capital Social: contenidos temticos

no la entendan necesariamente como aos ms tarde los seguidores como Putnam o Durston
la definen.

Mientras que Bourdieu trata inicialmente esta nocin de modo instrumental y, como dice
Portes1, "se concentra en los beneficios que reciben los individuos en virtud de su participacin
en grupos, y en la construccin deliberada de la sociabilidad con el objetivo de crear ese
recurso", lo que hace que el Capital Social pueda tener consecuencias negativas, como las
redes mafiosas por ejemplo, la tendencia actual es de centrarse en los aspectos positivos del
trmino e insistir en los provechos intrnsecos a la sociabilidad asociativa. Es obvio que nuestro
enfoque de tica, Capital Social y Desarrollo est en esa misma lnea.
Aunque sea difcil definir esta nocin de Capital Social, podemos discernir los siguientes rasgos
que establecen un consenso desde nuestro enfoque de tica, Capital Social y Desarrollo, segn
Enrique Garca, Presidente Ejecutivo de la Corporacin Andina de Fomento. Para este autor, el
Capital Social:

"Es concebido como el conjunto de normas de confianza, valores,


actitudes y redes entre personas e instituciones en una sociedad,
que define el grado de asociacin entre los diferentes actores
sociales y facilita acciones colectivas y de cooperacin.
El conjunto de normas de confianza, valores, actitudes y redes
Puede ser considerado capital en cuanto su utilizacin genera beneficios individuales y
colectivos. De acuerdo con Kliksberg (2003), los elementos que componen el capital social son
muestras de la riqueza y fortaleza del tejido social de una sociedad que permiten tener
beneficios a las personas y a la sociedad en su conjunto.
Entre personas e instituciones en una sociedad, que define el grado de asociacin entre
los diferentes actores sociales
Puede tener varias dimensiones: una individual, es decir el grado de integracin a su entorno a
partir de las relaciones ms cercanas de una persona, familia o empresa; una dimensin
sectorial, es decir la accin de personas, familias o empresas en su sector o entorno ampliado,
su interrelacin y su relacin con los poderes pblicos. Esto tiene que ver con la pertenencia a
redes comunitarias, gremios, asociaciones empresariales, etc. Finalmente, est la dimensin
colectiva o nacional, es decir el capital social entendido como un acervo de una sociedad en su
conjunto.
Facilita acciones colectivas y de cooperacin
Es til para el diseo de polticas pblicas con una ptica ms integral, multidisciplinaria y
sistmica."

A. Portes: Capital Social: Sus orgenes y aplicaciones en la sociologa moderna, en: "De igual a igual. El desafo del Estado ante
los nuevos problemas sociales. FLACSO/FCE, 2000.
2

Portal Educativo de las Amricas


http://www.educoea.org

Cmo ensear tica, Capital Social y Desarrollo en la Universidad?


Mdulo 4: tica y Capital Social: contenidos temticos

Puede ampliar la lectura sobre este tema consultando la biblioteca digital de la


Iniciativa en:
http://www.iadb.org/etica
L. Enrique Garca, Capital social clave para el desarrollo en Amrica Latina.
La idea primigenia del Capital Social es pues la de un beneficio
especficamente social ligado a la pertenencia solidaria a un grupo que
permite a sus miembros acceder a recursos y bienes inaccesibles sin la
existencia de dicho grupo. En otros trminos la definicin nuclear del
Capital Social sera la de una red de beneficios propia de la sociabilidad; la
calidad, cantidad e intensidad de las relaciones estructurando el Capital
Social en menor o mayor forma. Se trata de un capital en cuanto produce
recursos y riquezas, que se expresan en redes de beneficios, contactos y
ayuda recproca, que tienen su traduccin econmica posible, y sobre todo
facilitan la consecucin de objetivos en la vida social.
Como tal, el Capital Social permite una interesante traduccin del pensamiento clsico
econmico en trminos ms sociales y ticos: si las teoras clsicas de la economa son
demasiado individualistas, se concentran slo en los capitales econmicos y el capital humano
individual visto dentro de una relacin de competencia entre sujetos aislados y maximizadores,
la insistencia en el Capital Social permite complejizar la comprensin de las fuentes de riqueza
al introducir las variables culturales, simblicas y sociales. Estamos tpicamente delante de un
concepto trans e inter disciplinario que reincorpora la economa dentro de las ciencias sociales,
ella que con el hechizo de su matematizacin haba vivido la ilusin de su autonoma frente a
las dems dimensiones de la vida social. Intil de precisar que el enfoque de Capital Social
fuerza a un cambio profundo en la estructura curricular de los estudios econmicos, cambio que
no se ha dado todava con toda la nitidez que se necesitara.
Pero lo ms interesante de la nocin es que introduce dimensiones ticas y valorativas en la
produccin de capital, puesto que el elemento "facilitador" del Capital Social que permite el
acceso a nuevos recursos y beneficios depende explcitamente de lazos de confianza y
cooperacin solidaria. Por esta razn, John Durston 2 define el trmino haciendo referencia a
"las normas, instituciones y organizaciones que promueven la confianza, la ayuda recproca y la
cooperacin, lo cual trae tres tipos de beneficios: reducir costos de transaccin, producir bienes
pblicos y facilitar la constitucin de organizaciones de gestin de base efectivas, de actores
sociales y de sociedades civiles saludables". Definicin muy clara en descartar la red mafiosa
como posible ejemplo de Capital Social, pero definicin que debe ser precisada en el sentido en
que el beneficio producido excede lo que se podra conseguir a travs de una simple relacin de
intercambio entre annimos en el mercado3. Esto nos conduce a valorar las relaciones
sociales de solidaridad, cooperacin y confianza como productoras de una riqueza y
beneficio social sostenible (porque ligados con el mismo tejido social) que no podra ser
obtenido desde el mero mercado. As, no podemos ms atribuir relaciones econmicas el
producto de la riqueza de una nacin, sino tambin y ms originariamente a las mismas
relaciones sociales.
2

J. Durston: El Capital Social en 6 comunidades campesinas de Chile, CEPAL, Divisin de Desarrollo Social, 2000.
Ver tambin: http://www.asesorias.tie.cl/documentos.htm
3
Segn la perspectiva desarrollada en la Iniciativa de Capital Social de la Universidad de Michigan.
3

Portal Educativo de las Amricas


http://www.educoea.org

Cmo ensear tica, Capital Social y Desarrollo en la Universidad?


Mdulo 4: tica y Capital Social: contenidos temticos

Tambin puede consultar los trabajos publicados por Robinson, Schmid y


Siles en la biblioteca digital de la Iniciativa: www.iadb.org/etica

La novedad invalorable que nos aporta el concepto de Capital Social es, en ese sentido, la
posibilidad de demostrar lo absurdo que constituye, desde un estricto punto de vista econmico,
el hecho de destruir relaciones sociales y culturales reemplazndolas por meras relaciones
econmicas, al incentivar la mercantilizacin de todas las dimensiones de la vida humana y las
negociaciones comerciales entre desconocidos en competencia permanente, sistema
profundamente antisocial que nos hace creer que todas las relaciones son del tipo de los juegos
de suma nula: Si t ganas yo pierdo, si yo gano t pierdes. Al hacer esto, que muchos todava
llaman "el progreso", "la modernizacin", "el desarrollo", no slo la comunidad se priva de los
recursos propios del Capital Social (muchos de ellos invisibles para la contabilidad de la nacin
pero muy reales como son los actos gratuitos, voluntarios y las redes de facilitacin de acceso a
servicios y bienes o recursos), sino que marginaliza los sectores ms pobres al obligarles a
pagar por servicios que antes podan obtener gratuitamente dentro de sus redes de cooperacin
y reciprocidad4. Por eso, es fundamental controlar y acompaar la relacin econmica en el
mercado con una tica y responsabilidad social adecuadas en trminos de normas de confianza
interpersonales, compromiso cvico con una ley que cuide del tejido cultural, respete a las
diferencias y las excepciones que implican5 , promueva y proteja el tejido asociativo civil y las
estructuras sociales locales, para que las relaciones sociales no sufran de su "contaminacin"
por la lgica del mercado.
Por la ausencia de tal cuidado social y cultural en la mayora de los pases pobres, no es
exagerado hablar hoy, en nuestras naciones, de una nueva pauperizacin ligada a la "riqueza"
de la globalizacin de las relaciones econmicas a costa de las relaciones sociales. Tampoco
es exagerado relacionar las dificultades de los pases ricos para brindarles a sus ciudadanos los
servicios sociales pblicos con la prdida neta de Capital Social asociada a la individualizacin y
mercantilizacin de la vida cotidiana. Pero lo ms escandaloso desde un punto de vista tico fue
sin duda la dramtica poltica hiperliberal impuesta a los pases pobres en las ltimas dcadas y
que ha conducido a la "tercermundializacin" acelerada de poblaciones enteras, con el afn de
"integrar" los grupos sociales al Mercado global, destruyendo de paso organizaciones sociales y
sus mismos sistemas culturales a golpe de anomia, xodo rural, etnocidio, etc. No nos
olvidemos, como acadmicos, que en la raz de esta locura yace una "inteligencia ciega" y una
simplificacin y separacin a ultranza de la problemtica econmica de las dems dimensiones
de la humanidad, cosa que el enfoque del pensamiento complejo hubiera podido prevenir e
impedir. Volveremos sobre este "mal desarrollo" dentro de poco.

Como por ejemplo ser asistido en su vejez o al momento de morir, servicio que cuesta ahora una fortuna en los pases dichos
"desarrollados" pero vctimas de la atomizacin social y la soledad asistida por servicios pblicos o privados.
5
En ese sentido, la poltica francesa de "excepcin cultural" que protege del mercado a los bienes culturales y artsticos es sin duda
el factor que ha podido preservar el liderazgo cultural y artstico de esta nacin, cuando otros pases europeos han visto casi
desaparecer su produccin nacional bajo los golpes de la industria cultural globalizada.
4

Portal Educativo de las Amricas


http://www.educoea.org

Cmo ensear tica, Capital Social y Desarrollo en la Universidad?


Mdulo 4: tica y Capital Social: contenidos temticos

Tipos de Capital Social


Muchos autores sealan una fuerte relacin entre Capital Social y gobernabilidad democrtica,
al incentivar el empoderamiento de la sociedad civil para la vigilancia ciudadana. Para el nivel
local, Franulic menciona al respecto:
"Una comunidad con la presencia de una estructura fuerte de Capital Social es bastante beneficiosa para
las familias y las autoridades locales, ya que muchas de las prcticas sociales quedan inmediatamente
bajo la vigilancia y el control de los mismos miembros de la comunidad, conformando as un circuito
6
espontneo de reproduccin de las normas" .

Pero el enfoque de Capital Social es sobre todo til para redefinir las tareas polticas de una
democratizacin idnea de la sociedad a partir de la articulacin entre los niveles en que se
produce y reproduce dicho capital, desde el mbito interpersonal hasta el Estado. Este esquema
puede ayudarnos a visualizar los diferentes tipos o escalas de Capital Social que los
investigadores suelen distinguir:

Nivel precursor de Capital Social. Relacin interpersonal de afinidad y confianza


Nivel micro de Capital Social. Redes de cercanas informales entre pares
Nivel medio de Capital Social. Redes de instituciones formalmente reconocidas de
la sociedad civil.
Nivel macro de Capital Social. Estructura social del Estado, clima social de
confianza y participacin poltica de la sociedad civil.
A pesar de que se trate de una nocin por definicin social, podemos hablar de Capital Social
individual en el sentido del "crdito que ha acumulado la persona en la forma de reciprocidad
difusa que puede reclamar en momentos de necesidad, a otras personas para las cuales ha
realizado, en forma directa o indirecta, servicios o favores en cualquier momento en el pasado"
(Durston, op.cit.).

F. Franulic: La discusin sobre el Capital Social: distinciones conceptuales bsicas, Estudios Sociales 111, 2002, Corporacin de
Promocin Universitaria, Chile.
5

Portal Educativo de las Amricas


http://www.educoea.org

Cmo ensear tica, Capital Social y Desarrollo en la Universidad?


Mdulo 4: tica y Capital Social: contenidos temticos

Notemos sin embargo que el Capital Social individual no es algo que se posee como un bien
privado, sino que depende de una red local de cooperacin y solidaridad en la cual las
personas se conocen y reconocen en cuanto tal como pertenecientes a una misma comunidad
de confianza y cooperacin (redes egocentradas de vecindario, comunitarias, religiosas, etc.).
La persona puede "interceptar" los beneficios producidos socialmente como miembro de la red,
pero no como individuo aislado, calculador y maximizador. Es decir que la participacin en los
beneficios depende de ciertas normas solidarias que el individuo debe seguir y respetar para
pretender ser reconocido dentro del grupo. Como tal, el Capital Social individual en redes
egocentradas puede propiciar formas desviadas negativas de capital social, como son la mafia,
redes de corrupcin o contrabando, etc.
En cambio, el Capital Social comunitario, segn Durston: "consta de las normas y estructuras
que conforman las instituciones de cooperacin grupal. Reside, no en las relaciones
interpersonales didicas, sino en estos sistemas complejos, en sus estructuras normativas,
gestionadoras y sancionadoras" (dem.). Luego, el Capital Social comunitario se refiere a la
"institucionalizacin de las relaciones de cooperacin y ayuda recproca en el marco de
organizaciones, empresas, comunidades locales y grupos que conforman la sociedad civil". Se
trata de la sociedad civil organizada y formalizada en sus instituciones y redes de instituciones.
La ventaja del Capital Social comunitario es que permite producir bienes pblicos a gran escala
y para cualquier persona, sin necesidad, de cercana y relaciones interpersonales entre los
actores de la red.
Putnam, en su famoso estudio sobre las dos Italias del Norte y del Sur
(http://www.cambiocultural.com.ar/publicaciones/putnam.htm, ha introducido un nivel superior de
Capital Social nacional referido esta vez a ciertas normas y valores de los pueblos que
constituiran su "virtud cvica". Se trata aqu de un especie de clima social de responsabilidad
ciudadana y civismo que puede diferenciar una sociedad entera de otra en sus expectativas y
patrones de cooperacin y confianza. Como tal, este tipo de Capital Social se refiere, ms all
de la dimensin de tica cvica, al aparato jurdico y al buen funcionamiento del Estado, de sus
servicios pblicos y su capacidad de articular y dialogar con la sociedad civil. Las sociedades
ms desarrolladas y confiadas, con una buena institucionalidad de los servicios pblicos,
consiguen obviamente mejores resultados en este aspecto.
Le pedimos que reflexiones acerca de cmo podra definirse y verse
representado el concepto de Capital Social Individual y Comunitario
dentro de la universidad.

Capital Social, Economa y Democracia


Una encuesta mundial de la Universidad de Michigan sobre el tema de confianza interpersonal
realizada en 1995, arroj los siguientes resultados en los cuales se puede ver la dramtica
situacin de Amrica Latina en cuanto Capital Social nacional. A la pregunta si las personas son
en general confiables o si hay que ser cuidadoso, se obtuvieron los siguientes porcentajes:

Portal Educativo de las Amricas


http://www.educoea.org

Cmo ensear tica, Capital Social y Desarrollo en la Universidad?


Mdulo 4: tica y Capital Social: contenidos temticos

Grupo de pases

Confiables

Hay que ser


cuidadoso

Total

Europa Occidental /
USA

44.6

55.4

100.0

Ex socialistas

24.1

75.9

100.0

Amrica Latina (sin


Per)

16.6

83.4

100.0

Per

5.0

95.0

100.0

Promedio mundial

25.9

74.1

100.0

Fuente: Universidad de Michigan. Encuesta mundial de valores


(citado in: Valores democrticos y participacin ciudadana en el Per 1998-2001, Martn Tanaka y Patricia Zrate, 2002)

Los valores cvicos practicados en un pas, cuando encuentran una comprobacin emprica en
el buen trato social cotidiano que una persona puede esperar tanto en la calle, en las empresas
y en los servicios pblicos, crean un clima de intercambio sumamente provechoso para la
economa, porque se reducen los costos de transacciones (no necesidad de muchos controles
contra la estafa por ejemplo) y se dinamiza la iniciativa econmica como tal: no nos olvidemos
que el capital de confianza es un motor de la actividad econmica. As, el enfoque de Capital
Social crea una relacin ntima entre valores ticos, prcticas sociales y culturales y desarrollo
econmico.
Pero est calidad del entorno de sociabilidad que constituye el Capital Social como facilitador
del desarrollo, depende en gran medida de la relacin del Estado con la sociedad civil.
Fukuyama7 menciona que en las sociedades con baja confianza social y centradas en las
familias ms que en redes sociales amplias, se observa una caracterstica comn: todas
atravesaron un perodo de fuerte centralismo poltico. La centralizacin poltica suele destruir las
instancias de poder local y las redes comunitarias, restndole protagonismo y poder a la
sociedad civil, quedando el individuo y su familia bajo la dependencia exclusiva del Estado. Al
contrario, en las sociedades bien descentralizadas y con mucha libertad de asociacin y
organizacin civil, hay ms confianza entre las personas, base para relaciones econmicas de
cooperacin
y
desarrollo.
Desde luego, habra una relacin intrnseca entre el desarrollo econmico y la democracia
participativa con fuerte autonoma de la sociedad civil, como lo ha ido demostrando el Nobel de
economa A. Sen, lo que nos conduce a pensar un nuevo modelo de desarrollo desde el
enfoque de Capital Social en trminos de articulacin entre la sociedad civil y el Estado,
enfatizando la sostenibilidad cultural y social del crecimiento (que el crecimiento econmico no
se pague con una disminucin del Capital Social) y la calidad de la sinergia entre actores
socioeconmicos e instituciones del Estado. Se trata de crear las condiciones para el desarrollo
de un "Estado inteligente" como lo dice Kliksberg, que sea "promotor y facilitador de desarrollo
7

Fukuyama F.: "Capital Social y Economa Global", en : Revista colombiana de ciencia poltica, n40, 1995.
7

Portal Educativo de las Amricas


http://www.educoea.org

Cmo ensear tica, Capital Social y Desarrollo en la Universidad?


Mdulo 4: tica y Capital Social: contenidos temticos

de una sociedad civil cada vez ms articulada, fuerte y activa", un Estado con papel
"sinergizante permanente" (dem p 14).

Si desea ampliar su anlisis de este concepto puede consultar:


B. Kliksberg: Repensando el Estado para el desarrollo social, p 13:
http://www.iadb.org/etica/SP4321/DocHit.cfm?DocIndex=342

Esta nueva figura del Estado interlocutor y facilitador en la era del desarrollo global permite
resaltar la importancia entre las dos "direcciones" bsicas del Capital Social tal como el Banco
Mundial las define como8: asociaciones "horizontales" y asociaciones "verticales". Si la
horizontalidad define un Capital Social basado en redes sociales y normas asociadas entre los
individuos de una sociedad, la verticalidad se refiere a relaciones que trascienden las redes
particulares hacia el acceso a la formalidad y la articulacin con el Estado. Al evitar que la red
de cooperacin horizontal se torne en inters particular contra el bien comn, este punto de
vista bi-direccional: " no slo toma en cuenta las virtudes y los vicios del capital social, y la
importancia de la formacin de relaciones dentro y entre comunidades, sino tambin reconoce
que la capacidad de varios grupos sociales para actuar en su propio inters depende
crucialmente del apoyo (o la falta de apoyo) que reciban del Estado y del sector privado. De la
misma manera, el estado depende de la estabilidad social y del apoyo popular. En resumen, el
desarrollo econmico y social prospera cuando los representantes del estado, del sector
empresarial y de la sociedad civil crean foros a travs de los cuales puedan identificar y
alcanzar
metas
comunes."
La sinergia que se trata de crear puede representarse del siguiente modo:

Qu es lo que genera Capital Social?


En un primer nivel histrico y cultural, lo que produce espontneamente Capital Social es el
conjunto de normas y valores culturales compartidos, las historias y los smbolos colectivos
que una comunidad hereda y reproduce, que constituyen su "Nosotros", su ethos especfico.
Esta fuente cultural de Capital Social se trasmite mediante la socializacin temprana. Se trata
de una memoria social de identidad y etnicidad, incluyendo aspectos religiosos y relaciones de
parentesco.
8

Ver: Qu es el Capital Social?, www.worldbank.org/poverty/spanish/scapital/index.htm


8

Portal Educativo de las Amricas


http://www.educoea.org

Cmo ensear tica, Capital Social y Desarrollo en la Universidad?


Mdulo 4: tica y Capital Social: contenidos temticos

En un segundo nivel de la accin social, son los principios y prcticas de reciprocidad y


cooperacin que acumulan Capital Social en el sentido en que articulan a los individuos
alrededor de acciones e intereses comunes, creando hbitos de solidaridad y ayuda mutua. La
capacidad de asociarse presente en la sociedad civil de una nacin es un indicador importante
de Capital Social.
Finalmente, en un tercer plano propiamente poltico, la compensacin de las desigualdades
econmicas mediante servicios de seguridad social y redistribucin de la riqueza producida
ayuda en mantener el Capital Social de una nacin. Asimismo, el empoderamiento de la
sociedad civil organizada y el reconocimiento institucional de las organizaciones populares
locales crean las condiciones para una interlocucin sana entre el Estado y la sociedad civil.
As, la acumulacin y reproduccin de Capital Social estn relacionadas con factores histricos,
culturales, sociales y polticos que las polticas sociales empiezan a tomar ms en cuenta.
Franulic propone, en el artculo citado (p103-104), cuatro tipo de polticas para promocionar el
Capital Social:
1. Establecer polticas de promocin del enfoque de Capital Social: que exista "una
comunidad cientfica articulada en torno al enfoque y adems una infraestructura tcnica,
para la realizacin de investigacin acadmica, aplicada e intervenciones sociales, a nivel
de Universidades, ONGs, organismos intergubernamentales e instituciones estatales".
Esta poltica interesa en primer instancia la Responsabilidad Social Universitaria.
2. "Desarrollar una plataforma de polticas culturales que sirvan de base para el desarrollo de
experiencias precursoras de Capital Social () Una poltica consciente de promocin de la
cultura popular y campesina en Amrica Latina es un mecanismo muy adecuado para
fomentar la identidad local y as posibilitar la emergencia de Capital Social." Es muy
importante trabajar para la revaloracin de la autoestima comunitaria, sobre todo en
nuestro contexto histrico de colonialismo, etnocidio, discriminacin racial, cultural y de
gnero, que hacen muchas veces imposible para los pueblos latinoamericanos una
verdadera integracin nacional en un "Nosotros" comn9.
3. Enfocar la accin estatal en las "polticas de creacin y fortalecimiento de Capital Social
como forma de intervencin directa en la comunidad".
4. Desarrollar polticas de coordinacin de Capital Social, para articular los esfuerzos entre
focalizacin de las acciones locales de desarrollo emprendidas y creacin de redes y
polos de desarrollo regionales y nacionales, para aumentar la efectividad de los
programas. En efecto, Franulic resalta que: "una comunidad puede por medio de un
programa social lograr altos niveles de Capital Social, pero esas comunidades aisladas no
lograrn salir adelante solas en un mercado econmico altamente competitivo donde no
vale de mucho las confianzas y la cooperacin intracomunitaria". Por lo tanto, seala el
autor que los agentes estatales deben de "ayudar a generar una red de contactos para las
ventas", lo que constituye tpicamente un problema de asociacin "vertical" para el Capital
Social.

Ver sobre este tema N. Lechner: "El Capital Social como problema cultural", en : Revista mexicana de sociologa, Vol. 64, N2,
Abril-Junio, 2002.
9

Portal Educativo de las Amricas


http://www.educoea.org

Cmo ensear tica, Capital Social y Desarrollo en la Universidad?


Mdulo 4: tica y Capital Social: contenidos temticos

Redefiniendo el desarrollo desde el enfoque de Capital Social


La estrategia de desarrollo cambia entonces radicalmente cuando depende del enfoque de
Capital Social. En lugar de centrarse en las metas del crecimiento econmico o en los bienes y
servicios por producir desde una visin asistencial de la ayuda a los sectores desfavorecidos,
el eje estratgico se centra en los sujetos del desarrollo, sus capacidades asociativas y de
autoliderazgo para emprender acciones de cambio social.
Desde este enfoque de "Autodesarrollo" que hemos abordado en el Mdulo 3, podemos definir
el proceso de desarrollo social del siguiente modo:
El desarrollo social es una secuencia cumulativa y sostenible de sinergias sociales
exitosas y moralmente deseables, para la promocin de la autonoma de las personas.
Como "secuencia cumulativa y sostenible", el desarrollo no es un acto puntual sino una
transformacin global de una comunidad humana. No es un resultado momentneo sino una
dinmica que se nutre de s misma para ser cada vez ms capaz de garantizar futuros xitos.
Su xito no puede ser "a costa de" sino que debe ser sostenible en relacin con el entorno
ecolgico, humano, organizacional y cultural.
Como "sinergias sociales", el desarrollo no es un logro individual o algo que pueda ser
destructor de relaciones, antisocial, sino al contrario una dinmica producta y productora de
mejoras en las relaciones sociales, de convivencia ms armoniosa y articulacin de las diversas
energas de los actores sociales hacia un propsito comn. Como tal, el desarrollo significa
cohesin social y acumulacin de Capital Social.
Que estas sinergias sociales sean "exitosas y moralmente deseables" significa que los logros
concretos alcanzados producen consenso y legitimidad, son como tales "universalizables" para
la comunidad en cuestin, es decir reconocidos como buenos; no marginan sino que integran.
Por lo tanto, el desarrollo debe poder ser calificado como "progreso" real de la comunidad frente
a una situacin anterior menos deseable, menos universalizable.
Estas sinergias sociales deben articular en todo momento los aspectos propiamente
econmicos e instrumentales de satisfaccin de necesidades materiales con los aspectos
propiamente culturales y relacionales de satisfaccin de "necesidades espirituales e nter
subjetivas" como podramos llamarlas. En una muy interesante investigacin sobre participacin
social realizada en Chile10, las autoras mencionan lo siguiente:
"Con frecuencia se ha mencionado que la participacin social actual est marcada por una motivacin
instrumental. Contrariamente, este estudio observa el peso creciente que tiene la dimensin simblica de
la participacin social. Cul es la diferencia entre ambas? La lgica instrumental supone articular
esfuerzos colectivos en torno a la obtencin de beneficios materiales especficos que, en ocasiones,
vienen predefinidos desde la oferta pblica. La lgica expresiva, por su parte, tiene que ver con la
capacidad de generar espacios de encuentro colectivos en los que la propia subjetividad se pone en
juego y es reconocida.

10

Ver: Claudia Serrano y Daniela Vicherat: Qu motiva a la gente a actuar en comn? Estudio sobre Participacin Social en la
Poblacin Malaquas Concha Comuna de La Granja, CIEPLAN Diciembre, 2000 http://www.asesorias.tie.cl/La_Granja.rtf.
10

Portal Educativo de las Amricas


http://www.educoea.org

Cmo ensear tica, Capital Social y Desarrollo en la Universidad?


Mdulo 4: tica y Capital Social: contenidos temticos

Tanto entre los grupos de corte tradicional como entre los emergentes, los grupos de corte puramente
instrumental, cuyo mximo exponente son los comits de pavimentacin, tienen una duracin acotada y
estructurada a los propsitos del comit, lo que a la larga significa su extincin. De esta forma, lo que
permanece en el panorama social comunitario es otro tipo de grupos, menos ligados a la coyuntura,
aunque roten los miembros: los grupos de mujeres, jvenes y adultos mayores, juntas de vecinos y otros.
Hasta hace pocos aos tenda a ubicarse a los jvenes y grupos culturales como representantes casi
nicos de la lgica expresiva. Sin embargo, la fuerza con la que han emergido los clubes de adulto
mayor, que funcionan en su mayora como espacios de expresin y acogida, as como la orientacin de
los grupos de mujeres hacia el desarrollo de la autoestima y habilidades sociales, dan cuenta de la
relevancia de los aspectos expresivos y simblicos, relacionados con las identidades y problemticas
especficas de los sujetos.
Lo anterior no contradice el peso de los intereses prcticos que subyacen a los comportamientos
asociativos y la fuerte demanda por obtener logros concretos. Lo que se observa es que los grupos que
se consolidan en el tiempo, necesariamente asumen una combinacin entre ambas lgicas, en la medida
en que logran constituirse como canales para la materializacin eficiente de beneficios a sus miembros,
pero a la vez son capaces de acogerlos, ser grupo de referencia y pertenencia y representarlos frente a
los otros.
En la bsqueda de categoras que permitan exponer como se expresa la dimensin simblico-afectiva de
la participacin en grupos y organizaciones en el nivel comunitario, detectamos cuatro reas que explican
la importancia de la experiencia grupal para las personas. Todas ellas tienen en comn el rol central que
adquiere la dimensin del sujeto y el espacio que se reclama para el Yo. En primer lugar se sita un rea
relacionada con la proteccin, la compaa y los sentimientos de seguridad y aceptacin que el mbito
grupal genera. Luego se ubican el rea formativa y de aprendizaje, situada en el contexto del desarrollo
de las personas. El tercer mbito dice relacin con el esparcimiento y la recreacin que representa el
encuentro con otros mientras el cuarto habla de la dinmica grupal como un espacio para uno mismo."

Entonces, el xito de un programa social estara ligado al hecho de poder generar y mantener
sinergias sociales en torno a necesidades instrumentales y simblico-afectivas, tejiendo juntos
la lgica econmica con la lgica cultural y subjetiva, para que los beneficios producidos no slo
signifiquen una mejora material, sino tambin anmica y de vnculo social.
Un "ejemplo ejemplar", si podemos decir, de este tipo de enfoque de Capital Social para el
desarrollo es el ya famoso programa costarricense de los Tringulos de solidaridad.
Le

proponemos

que

lea

los

siguientes

textos

de

1. tica y economa: la experiencia del tringulo


http://www.iadb.org/etica/SP4321/DocHit.cfm?DocIndex=278

Astrid

de

2.
el
triangulo
de
la
solidaridad,
una
experiencia
http://www.iadb.org/etica/SP4321/DocHit.cfm?DocIndex=1095

la

Fischel:
solidaridad

innovadora

En qu medida este ejemplo de estrategia de desarrollo ilustra un modo tico de


concebir el desarrollo?

11

Portal Educativo de las Amricas


http://www.educoea.org

Cmo ensear tica, Capital Social y Desarrollo en la Universidad?


Mdulo 4: tica y Capital Social: contenidos temticos

Qu indican acerca del ethos poltico y social de nuestros Estados actuales las
resistencias encontradas por A. Fischel para desarrollar los Tringulos de
solidaridad?
Comparta algunas de sus conclusiones con sus compaeros de
Foro

Qu es lo que genera Capital Antisocial?


Una buena manera de hacer entender a los estudiantes lo que es el Capital Social es de
reflexionar con ellos acerca de lo que podramos llamar el "Capital Antisocial", es decir la
acumulacin de prdidas sociales de cooperacin relacionadas con la desconfianza, el
etnocidio, el "pleitismo jurdico", los valores antisolidarios, la desintegracin social, etc.
Si, como lo hemos visto, Amrica Latina se encuentra muy por debajo del promedio mundial en
lo que concierne el grado de confianza social sentida en la Encuesta mundial de valores de la
Universidad de Michigan, deben de existir razones sistmicas que explican esta situacin.
Vamos a resaltar slo tres, en relacin con:
1. En primer instancia, es obvio que el aumento de la pobreza y de la desigualdad en
nuestro continente, el ms desigual del mundo, es una cruda y cruel realidad de las
ltimas dcadas, que incide en el desarrollo del Capital Antisocial, al crear abismos de
diferencias entre las clases sociales y entre las personas11. Esta situacin estructural se
reproduce en todas las esferas de la vida humana, y sobre todo en la educacin. Si en
muchos pases de Europa occidental el 20% ms rico de la poblacin gana slo entre 3 y 4
veces ms que el 20% ms pobre, y todos los nios (ricos o pobres) pueden encontrarse
en las mismas escuelas pblicas de buena calidad, la situacin en Amrica Latina es muy
diferente. No slo es casi imposible en pases como el Per o Brasil que el hijo de rico se
cruce en la escuela con aquel del pobre, sino que en 1998, segn cifras del BID, los jefes
de hogar del 10% de ingresos ms altos de 15 pases de la regin tenan 11,3 aos de
educacin promedio, mientras que el 30% ms pobre slo 4,3 aos (Kliksberg, op.cit.).
Intil es recalcar cmo est desigualdad educativa se auto reproduce y crea finalmente una
brecha social imposible de disminuir. Llegados a estos extremos niveles de fragmentacin
social, en los cuales las personas no han compartido nunca la misma experiencia social, ni
aprendido las mismas lecciones escolares, ni siquiera en muchos casos el mismo
lenguaje12, no queda ningn mbito del espacio pblico en el cual puedan presentarse los
diversos "mundos" socioculturales en pie de relativa igualdad. La universidad,
desgraciadamente, no escapa a esta lgica de exclusin y fragmentacin social.
2. Ms all de este aspecto estructural, el clima de desconfianza que contamina el continente
est tambin ligado a la falta de una verdadera interlocucin entre el Estado y la
sociedad civil. Los problemas encontrados por A. Fischel en la implementacin poltica de
los Tringulos de solidaridad en un pas sin embargo con buenos resultados de integracin
11

Ver: B. Kliksberg, Diez falacias sobre los problemas sociales de Amrica Latina, op.cit.
http://www.clad.org.ve/congreso/klikdiez.html
En Lima, por ejemplo, las clases altas suelen mandar sus pequeos hijos en nidos donde se les ensea el ingls, requisito para
ser aceptados despus en los mejores colegios de la ciudad, mientras que en muchas provincias del Per, el mismo castellano es el
segundo idioma de los nios.
12

12

Portal Educativo de las Amricas


http://www.educoea.org

Cmo ensear tica, Capital Social y Desarrollo en la Universidad?


Mdulo 4: tica y Capital Social: contenidos temticos

social frente a otros de la regin, son testigos del mantenimiento de una mentalidad poltica
poco seducida por el dilogo social y el empoderamiento ciudadano. Si sumamos a esta
falta de actitud de dilogo y al comportamiento "verticalista" autoritario por parte del
funcionario, la mediocridad de los servicios pblicos brindados en Estados con escasos
recursos sociales, llegamos a niveles de suma desconfianza y descontento entre los
ciudadanos y lo que se supone es "su" administracin pblica, pero que muchas veces es
vista como un poder exterior, poco til y prepotente.
Una interesante investigacin cualitativa realizada para determinar cmo se percibe y auto
percibe el servidor pblico en el Per (pas con slo un 5% de confianza interpersonal segn la
encuesta de la Universidad de Michigan citada), arroja los siguientes resultados13:
"en trminos generales aparece como denominador comn un deterioro progresivo de la imagen del
trabajador pblico, deterioro que se ha ido incrementando con el tiempo, experimentndose una prdida
de valoracin de su rol y condicin en la sociedad." (p 48)

En cuanto a la auto percepcin del servidor pblico, los funcionarios se quejan de que estn
expuestos a la crtica permanente de los ciudadanos. Aparece la expresin de frustracin e
incluso resentimiento en tanto se percibe una evidente carencia de respaldo de parte del
Gobierno Central. El servidor pblico est disconforme con su remuneracin y las carencias de
infraestructura y logstica. Los funcionarios de mayor nivel perciben que lejos de ubicarse en un
lugar privilegiado estn sujetos a una doble presin, considerando que son objeto de mayores
crticas por parte de la sociedad, as como estn sujetos a un mayor control o fiscalizacin por
parte de las entidades en las que trabajan. Al respecto, mencionan que los medios de
comunicacin contribuyen de modo importante a propagar y generalizar una mala imagen del
sector pblico14.
Los ciudadanos, por su parte, tienden a percibir al trabajador pblico en forma ambivalente,
aunque predominan los aspectos de connotacin negativa. Se adjudica a los empleados
pblicos:
Un bajo nivel de eficiencia en el desempeo de su funcin (relajados, flojos, lentos, etc.), una
actitud displicente hacia el pblico (malcriados, indiferentes): "Nosotros les pagamos el sueldo y
ellos actan como si nos estuvieran haciendo un favor" (ama de casa). Tambin se les adjudica
un recurrente ejercicio de la corrupcin y se les percibe como personas con escasa preparacin
acadmica. Se les adjudica ser conformistas y mediocres en tanto carecen de aspiraciones (ley
del mnimo esfuerzo).
Entre los ciudadanos se percibe tambin que al interior de las entidades pblicas predomina la
filosofa del "sobe y raje": comportamientos orientados a conservar el puesto, con adular y
complacer a los superiores (sobe) y criticar y desprestigiar a otros colegas (raje) para ocupar
sus puestos. No obstante, la situacin del empleado pblico suscita cierta deseabilidad entre los
ciudadanos comunes, que perciben que gozan de una situacin "privilegiada" (ingreso fijo,
estabilidad laboral, beneficios de ley). Suscita luego envidia y clera, puesto que, segn la
opinin de la gente, "no merecen lo que tienen".

13

Ver: Precariedad y proyecto, tica y funcin publica en el Per. Escuela Superior Antonio Ruiz de Montoya y Defensora del
Pueblo, Lima Per, 2002.
14
El Per est saliendo de 10 aos de rgimen autoritario y corrupto del llamado "fujimontesinismo" y los medios de comunicacin
estn muy atentos a cualquier caso sospechoso de corrupcin
13

Portal Educativo de las Amricas


http://www.educoea.org

Cmo ensear tica, Capital Social y Desarrollo en la Universidad?


Mdulo 4: tica y Capital Social: contenidos temticos

Con respecto a quienes desempean puestos de direccin, gozan de reconocimiento y status


social, sobre todo en la provincia, lugares en los que mayormente se les ve como figuras de
autoridad y se les atribuye poder debido a su capacidad de decisin, pero tambin inspiran
cierto temor, lo que indica una relacin asimtrica de inferioridad hacia ellos. Se percibe que los
funcionarios de ms alto nivel tienden a permanecer un tiempo limitado en sus puestos, puesto
que se tratan de cargos de confianza. Esto crea una visin de marcada inestabilidad en los
puestos de direccin: "los directores son cambiados muy rpido, como cambia un foco y eso
malogra su gestin" (dice un servidor pblico).
Intil de precisar que en estas condiciones, es muy difcil esperar el desarrollo espontneo de
valores de cooperacin y confianza entre el servidor pblico y la poblacin civil. El Capital Social
es un bien frgil que es muy difcil reconstituir una vez perdido. En este contexto tico
dramticamente adverso, el actual gobierno peruano intenta una reforma del Estado basada en
un proceso de democratizacin y descentralizacin. Es el noveno intento de descentralizacin
de la historia republicana del Per.
Le proponemos que reflexione sobre estos interrogantes:

Cmo se expresa esta acumulacin de capital antisocial en tu propio pas?

Piensas que los estudiantes estn conscientes de estos problemas?

Ensear el Capital Social de modo ldico


A nivel pedaggico, puede resultar til para hacer percibir a los estudiantes en detalle la
problemtica del Capital Social mostrarles la importancia fundamental de la estructura de
una red para la generacin de Capital Social o de Capital Antisocial. Es obvio que la falta
de dilogo y el autoritarismo, cuando estructuran las redes organizacionales sociales, crean
disfunciones y desconfianzas que afectan el Capital Social y la accin colectiva de una nacin.
Para ensear a los estudiantes la importancia del diseo de una red de relaciones, una pequea
dinmica grupal utilizada como ejercicios en las escuelas de teatro, puede servir de ilustracin
de los problemas que las personas encuentran cuando la organizacin de sus redes est mal
pensada:
Aqu le proponemos una actividad que le permitir trabajar con sus alumnos e incluso con sus
colegas:

Se libera el espacio del saln de clase y se pide a los alumnos ponerse de pie, formar
una ronda, dndose las manos y cerrar los ojos. El animador (que forma tambin parte
del crculo) explica que el juego consiste en hacer circular una informacin bajo la forma
de un "apretn" de mano, que slo l puede generar, las dems personas en el crculo
solamente tienen que transmitirlo del siguiente modo: cuando la persona siente que se
le aprieta la mano derecha, "transmite" el apretn hacia su izquierda y viceversa. As,
idealmente, un apretn de mano debera circular de mano en mano indefinidamente.
Al empezar el juego (a ojos cerrados!) se le aconseja al animador que decida "mandar"
un apretn hacia su izquierda y otro hacia su derecha, para ver si los dos "vuelven" al
14

Portal Educativo de las Amricas


http://www.educoea.org

Cmo ensear tica, Capital Social y Desarrollo en la Universidad?


Mdulo 4: tica y Capital Social: contenidos temticos

completar el crculo. En general, se multiplican rpidamente los errores de transmisin,


los apretones se pierden o se multiplican sin razn, creando una alegra general.

En un segundo momento, a fin de "mejorar la comunicacin", se pide a los participantes


abrir los ojos, quedarse en crculo, y ya no hacer circular apretones sino levantar los
brazos hacia los costados, codos doblados y manos giradas hacia los costados para
mantenerlas palmas contra palmas con los vecinos de la derecha y la izquierda. En esta
posicin, se trata de transmitir leves "empujones" con la misma regla: slo el animador
puede generar un empujn y quien lo recibe por una mano lo tiene que transmitir en
seguida por la otra. A pesar de la posibilidad esta vez de "observar" el desplazamiento
del empujn y "controlar" las fallas de los compaeros, es bien difcil que un grupo no
experimentado logre transmitir mucho tiempo sin errores los empujones del animador
que, claro est, aumentar la dificultad agregando empujones si ve que el grupo logra
una correcta transmisin durante varias vueltas. Aqu tambin se logra rpidamente una
hilaridad general!

Finalmente, en un tercer momento, se pide a los participantes caminar libremente en el


espacio cuidando de repartirse del modo ms uniforme posible en l, sin dejar "huecos",
lugares no transitados. As, las personas empiezan a moverse fijndose en los
desplazamientos de los dems. Una vez que se logra este primer paso, se pide que
cualquier persona que lo desee se detenga de repente, sin previo aviso, quedando
totalmente "congelada" y se agrega la consigna de que cualquiera que vea una persona
"congelada" tendr que congelarse instantneamente tambin. Cada vez que todo el
mundo est "congelado" porque alguien habr tomado la decisin de detenerse, el
animador da una seal para empezar de nuevo a caminar, hasta que otra persona
decida congelarse a su vez. En menos de un minuto, en general, se logra un perfecto
nivel de escucha y armona entre los participantes, quedando el grupo totalmente
congelado en un segundo, sin que sea realmente posible saber quin se detuvo primero.

Despus, se procede a la interpretacin de la simulacin, atrayendo la atencin de los


estudiantes en los siguientes puntos:
En los dos primeros momentos (el apretn y el empujn) la red de transmisin
es de tipo lineal, es decir que existe un solo camino para ir de un punto a otro,
debiendo pasar por todos los puntos intermedios.
1
y
2

Al contrario, durante el ejercicio de la "congelacin", la red es de tipo complejo,


con muchos caminos posibles y ninguna posicin privilegiada entre los puntos.
Todos los puntos estn en interrelacin posible con todos los dems en
cualquier momento. Se parece al entretejido de un cerebro:

15

Portal Educativo de las Amricas


http://www.educoea.org

Cmo ensear tica, Capital Social y Desarrollo en la Universidad?


Mdulo 4: tica y Capital Social: contenidos temticos

Esta estructura de red permite transmitir la informacin con mucha velocidad y


eficacia, logrando una armona entre todas las partes.

Al final, se hace reflexionar los estudiantes sobre el tipo de redes que suelen percibir en su
entorno, y si la estructura del Estado en particular les parece compuesta de redes complejas o
lineales. Permite generar interesantes debates sobre la burocracia, el despilfarro de energa
gastado en mantener redes de comunicacin inadecuadas entre las personas de una
organizacin, etc.
Le proponemos que: juegue este juego con sus estudiantes, despus de haber
planteado el tema del Capital Social y organizar despus un debate sobre el mejor
diseo de una poltica social eficiente entre el Estado, el sector empresarial y la
sociedad civil organizada.
Despus de haber planteado el tema de Capital Social y desarrollo a los
estudiantes, organizar un debate a partir del proverbio Bant: "La fuerza del
cocodrilo es el agua".

La contradiccin: modernizacin y mal desarrollo


Una vez que los estudiantes estn en capacidad de relacionar tica, Capital Social y Desarrollo,
nos parece importante radicalizar el asombro y la visin crtica del desarrollo tal como se
entiende y practica actualmente, gracias a una reflexin crtica sobre el proceso de
modernizacin del mundo y su relacin con la produccin de "Capital Antisocial". Es tiempo,
pues, de ahondar en la nocin de Mal desarrollo.
A continuacin vamos a analizar cuatro aspectos claves de la crtica a la modernizacin del
mundo, que son de mucha utilidad para nuestro enfoque de Etica, Capital Social y Desarrollo:
El advenimiento de una nueva figura filosfica del ser humano considerado como
sujeto

El fenmeno social del etnocidio y la uniformizacin de los modos de vida

La promesa moderna no cumplida de brindar autonoma a las personas

Las patologas sociales relacionadas al consumismo.

Todos ellos nos permitirn entender la nocin de "Mal desarrollo".

16

Portal Educativo de las Amricas


http://www.educoea.org

Cmo ensear tica, Capital Social y Desarrollo en la Universidad?


Mdulo 4: tica y Capital Social: contenidos temticos

8.1. El sentido filosfico de la modernidad: advenimiento del sujeto


Entendemos el proceso de modernizacin como una ruptura en el mundo tradicional y el
nacimiento de un nuevo paradigma filosfico con implicancias fundamentales a nivel prctico: el
paso de una concepcin del ser humano en trminos de "criatura" incluida en un mundo creado
que lo ubica y le da su sentido a una concepcin del ser humano en trminos de "sujeto" que
se auto posiciona frente a un mundo objetivado al cual le da l su sentido. La relacin con el
mundo se ve entonces totalmente invertida. El ser humano criatura (concepcin tradicional) es
el resultado de un proceso de creacin que lo antecede y lo llama a ser quin es. Mientras que
el ser humano sujeto (concepcin moderna) es el inicio de un proceso de autora
indeterminado, una apertura temporal sin fin preestablecido, y sin lugar predispuesto. El ser
humano criatura consigue el sentido de su vida en su inclusin en un mundo de por s dotado
de sentido, concibe la existencia en trminos de vnculo con el Ser y de "re-ligacin" con l
(Religin). Mientras que el ser humano sujeto se relaciona de modo asimtrico con un mundo
de objetos a disposicin de su actuar propio. El mundo para l ha sido y debe ser siempre
objetivado, de tal forma que en su correcta objetivacin el sujeto consiga el sentido de su
existencia, construyendo un mundo para s mismo.

Lo que es importante subrayar, para no confundir la "modernidad" con una serie de


descubrimientos y proezas tecnolgicas como se suele pensar, es el hecho de que es un
cambio de paradigma en la percepcin del ser humano y de su mundo circundante (reducido al
estado de objeto) que ha permitido empezar a utilizar los descubrimientos tcnicos para
transformar el mundo en forma acelerada tal como se dio en los ltimos siglos, primero en
Europa occidental. Los pases de medio y extremo Oriente podan quizs tener un mayor nivel
de conocimiento cientfico y tcnico en el alba de la modernizacin del mundo, pero les faltaba
fundamentalmente la concepcin subjetiva del ser humano y la objetivacin del mundo.
A partir de este cambio paradigmtico fundamental de la concepcin del ser y de la existencia
humana, se puede entender la velocidad de transformacin de un mundo ahora por primera vez
reducido a una serie de objetos manipulables: las revoluciones industriales; y la transformacin
paralela de la visin del ser humano como individuo libre y sujeto de derechos universales e
inalienables: los derechos humanos. Pero tambin se puede entender las crisis de este nuevo
mundo humano, la "desubicacin" fundamental del individuo arrojado en un entorno
"desencantado": la crisis postmoderna; y tambin los grandes peligros que tiene que enfrentar
ahora una humanidad sujeta a su propia historia subjetiva, con un poder de destruccin
equivalente a su poder de creacin.

17

Portal Educativo de las Amricas


http://www.educoea.org

Cmo ensear tica, Capital Social y Desarrollo en la Universidad?


Mdulo 4: tica y Capital Social: contenidos temticos

Hacer el balance de estas dos consecuencias modernas es un ejercicio elemental de reflexin


para el ciudadano actual, que no se puede dejar engaar por las falsas promesas de la
publicidad comercial segn la cual todo lo nuevo es mejor.
Le proponemos que evale en qu medida la promesa de la
modernidad, que es de hacer de los seres humanos los sujetos libres
y autnomos de su propia historia y desarrollo, ha sido y es cumplida
o no, y cmo podramos actuar colectiva y responsablemente para que lo
sea, desde nuestro enfoque de tica, Capital Social y Desarrollo.

Vamos a abordar algunos temas que tienen directa relacin con el crecimiento del Capital
Antisocial en la vida cotidiana moderna.

8.2. El etnocidio: sistemas expertos y estandarizacin de la vida cotidiana


Una de las grandes caractersticas de la modernizacin de la vida cotidiana de las poblaciones
reside en la institucionalizacin cada vez mayor de sistemas expertos 15 para regular la
actividad social. Se trata de organizaciones especializadas en brindar servicios a partir de
conocimientos racionales de expertos que definen y norman los comportamientos que los
usuarios deben mantener. Por ejemplo, la aviacin civil es un sistema experto que regula el
servicio de transporte areo y determina el comportamiento adecuado de los pasajeros segn
normas racionalmente definidas e institucionalizadas por ley. Un sistema experto trata siempre
de inducir en la poblacin comportamientos racionales que debemos absolutamente distinguir
de lo que son las costumbres culturales.
La costumbre cultural es un fenmeno histrico que se relaciona con los sentidos inventados
por una comunidad para significar su mundo. Una costumbre es una norma social vivida,
significativa para esta comunidad, y diferenciada de las costumbres de las dems comunidades
(la costumbre fija la definicin de un nosotros particular que permite distinguir una cultura de
otra). Mientras que un sistema experto, basado en una ciencia, produce conocimientos
universales. Desde luego, los comportamientos deducidos de estos conocimientos sern
tambin comportamientos universales. Por ejemplo, el sistema experto de la higiene pblica
deduce normas urbansticas y prcticas individuales universalmente recomendables a partir de
conocimientos de la medicina bacteriolgica. Las reglas de higiene son universales desde el
punto de vista de la bacteriologa.
Se puede observar cmo el Sujeto pensante experto define el sentido de la realidad tal como
debe ser universalmente objetivada a partir de su propio pensar racional. La aceleracin de los
cambios en nuestra vida cotidiana se deben, en gran medida, a la actividad permanente de
correccin y mejoramiento que los sistemas expertos realizan, empujados por la competencia,
los descubrimientos cientficos, y la crtica social. La realidad vivida de las convenciones
sociales, las normas y los comportamientos, est constantemente sometida a los cambios
operados por los sistemas expertos, que reemplazan las costumbres culturales particulares
por comportamientos universales racionalmente inducidos.
15

Ver GIDDENS, A. Consecuencias de la Modernidad, Alianza Editorial: Madrid, 1993. El autor define los sistemas expertos como
sistemas de logros tcnicos o de experiencia profesional que organizan grandes reas del entorno material y social en el que
vivimos. (p. 37).
18

Portal Educativo de las Amricas


http://www.educoea.org

Cmo ensear tica, Capital Social y Desarrollo en la Universidad?


Mdulo 4: tica y Capital Social: contenidos temticos

Los sistemas expertos tienden as a homogeneizar y estandarizar los modos de vida de la


poblacin moderna: los sistemas escolares, de salud, administrativos, de comunicacin, de
produccin industrial, etc. van a evolucionar a partir de una misma manera de operar la
objetivacin de su realidad y problemas, aunque estn ubicados en China, Alemania, o
Ecuador. Por eso, los sistemas expertos van a entrar en conflicto con las costumbres
tradicionales de una comunidad, tratando de nivelarlas en comportamientos adecuados al
funcionamiento de la lgica del sistema.
Se trata del fenmeno que el filsofo contemporneo Jrgen Habermas llama la
contaminacin de la vida cotidiana por los sistemas abstractos. Nuestra vida cotidiana, la
pasamos ahora, sin darnos cuenta muchas veces, en relacin con miles de especialistas,
expertos, sistemas de organizacin y produccin, conocimientos cientficos. Detrs de cada
herramienta tecnolgica o servicio que usamos est un sistema experto: utilizar servicios
higinicos, pedir un prstamo bancario, manejar una computadora, viajar de vacaciones, etc.
son actos cotidianos que significan hoy hacer uso, como consumidor ignorante y confiado, de
los servicios y productos diseados por expertos segn conocimientos racionales, normados y
respaldados por leyes jurdicas y reglas administrativas, que determinan finalmente qu
comportamientos adecuados debe conocer y seguir el usuario. Siendo el conocimiento
racional y universal, el comportamiento ser tambin universal y racional16. Por lo tanto, los
sistemas expertos van contaminando nuestra vida y erosionando nuestras diferencias
culturales de interpretacin del mundo y actitud social, para terminar transformndonos en
usuarios estandarizados en todo el planeta de los mismos servicios uniformizados.
Un sistema experto evoluciona y se mejora a s mismo permanentemente, separndose de los
hbitos tradicionales, e inventando nuevos modos ms racionales de hacer las cosas. Estos
comportamientos racionalmente definidos son desarraigados de cualquier cosmovisin cultural,
de cualquier tradicin: no son ms que la deduccin de una serie de acciones objetivamente
pensadas por el sistema, segn su modo de operar la objetivacin de la realidad para resolver
problemas especficos (higiene, transporte, aprendizaje, comunicacin, etc.). Es as como el
proceso de modernizacin del mundo, a travs de la generalizacin de organizaciones
racionales de las actividades (los sistemas expertos), desemboca naturalmente en un vasto
proceso de etnocidio17, es decir de destruccin de las comunidades culturales particulares, en
sus costumbres, tradiciones y hbitos cotidianos propios. El etnocidio est ligado al proceso
de transformacin de las costumbres culturales particulares en comportamientos
uniformes inducidos por sistemas expertos.

16

No hay que confundir por supuesto esta universalidad con aquella de la ley moral. Obedecer a sistemas expertos no es un deber!
El trmino etnocidio (literalmente la matanza de una etnia) ha sido popularizado por el etnlogo Robert Jaulin, que lo define como
el acto de destruccin de una civilizacin, el acto de descivilizacin, que desorganiza la cotidianeidad de una comunidad en sus
relaciones de residencia, produccin, consumo y amor, en nombre del progreso de un ideal de humanidad nica definida en
trminos occidentales.
17

19

Portal Educativo de las Amricas


http://www.educoea.org

Cmo ensear tica, Capital Social y Desarrollo en la Universidad?


Mdulo 4: tica y Capital Social: contenidos temticos

Hablamos mucho en los ltimos tiempos de los riesgos de empobrecer la diversidad biolgica
de los sistemas ecolgicos del planeta. Se denuncia mucho menos el empobrecimiento en
diversidad cultural que significa la destruccin, violenta o suave, de etnias, culturas y
civilizaciones enteras en nombre del Desarrollo. Cada vez que muere una cultura, es un
idioma, una cosmovisin, una larga historia de conocimientos y sabidura, lo que Nietzsche
llamaba una posibilidad de vida que desaparece, empobreciendo nuestro modo de vida cada
vez ms uniformizado.
A propsito de este tema, el filsofo y economista C. Castoriadis escribe:

Se sabe que sobre este planeta, en el curso de miles de millones de aos, se despleg un biosistema
equilibrado compuesto por millones de especies vivas diferentes y que, durante cientos de milenios, las
sociedades humanas consiguieron crearse un hbitat material y mental, un nicho biolgico y metafsico
alterando el medio ambiente sin daarlo. A pesar de la miseria, la ignorancia, la explotacin, la
supersticin y la crueldad, esas sociedades consiguieron crearse a la vez modos de vida bien adaptados
y mundos coherentes de significaciones imaginarias de una riqueza y de una variedad sorprendentes.
Dirijamos la mirada hacia el siglo XIII, pasemosla desde Chartres a Borobudur y de Venecia a los
Mayas, de Constantinopla a Pekn y de Kublai-Kan a Dante; de la casa de Maimnides en Crdoba hasta
Nara y de la Carta Magna hasta los monjes bizantinos que copiaban a Aristteles; comparemos esta
fantstica diversidad con la situacin presente del mundo, en la que los pases no difieren realmente los
unos de los otros en funcin de su presente el cual, como tal, es el mismo en todas partes- sino
18
solamente en funcin de los restos de su pasado. Esto es el mundo desarrollado .

Desde luego, la contradiccin aparece en el hecho de que los sistemas expertos, que se
desarrollan en la Modernidad para controlar, racionalizar y mejorar la actividad social planetaria,
de modo solapado participan de una destruccin paulatina de las tradiciones y referentes
culturales particulares que permiten a una comunidad regularse a s misma e incluirse en su
entorno. En efecto, los significados culturales que constituyen una tradicin, son significados
ejes que permiten el consenso y la reproduccin social en largos perodos llamados
Civilizacin, y que son la materia prima del Capital Social, como lo hemos visto. Al contrario,
los comportamientos inducidos por sistemas expertos evolucionan constantemente, debido a los
avances y cambios cientficos y tecnolgicos, y slo conciernen un campo parcial,
especializado, de la actividad social, sin articulacin con los dems campos. Luego, los
sistemas modernos de control de la vida social son, pues, potencialmente generadores de
etnocidio, prdida de capital social, y anomia.
El crculo vicioso del etnocidio que acompaa el proceso de modernizacin del mundo,
es el siguiente:

18

Cornelius Castoriadis: Reflexiones sobre el Desarrollo y la Racionalidad, en: Colombia: el despertar de la Modernidad, F.
Viviescas y F. Giraldo Isaza (compiladores). Foro Nacional por Colombia (pp. 108-109).
20

Portal Educativo de las Amricas


http://www.educoea.org

Cmo ensear tica, Capital Social y Desarrollo en la Universidad?


Mdulo 4: tica y Capital Social: contenidos temticos

Esta problemtica del etnocidio debe ser examinada con mucho cuidado si queremos instituir un
nuevo y mejor modelo de desarrollo basado en el enfoque de Capital Social. No es un azar si
los proyectos participativos de los Tringulos de solidaridad evocados por A. Fischel cuestan
efectivamente mucho menos que los realizados por expertos. Tampoco es un azar si los
esfuerzos actuales de control de la evolucin social mediante sistemas administrativos expertos
no logran los propsitos buscados. Es que no se ha probado todava a escala de una nacin la
solucin participativa a los problemas sociales, se sigue insistiendo en un desarrollo
heternomo que resulta muy costoso, poco eficiente, y en contradiccin tica con la promesa de
autonoma de la era moderna.
8.3. La confiscacin moderna de la autonoma
El problema existencial del individuo moderno es que su nueva libertad le ha sido entregada
por las condiciones de vida ms que l haya sido y sea su protagonista. Sigue siendo para
cada uno de nosotros un reto volvernos los autores responsables de nuestra propia existencia
como "sujetos" en un mundo moderno dominado por la urgente necesidad de dinero y la
escasez de trabajo. Podemos definir el movimiento que subvierte la conquista moderna de
autonoma subjetiva como una objetivacin del mundo y de los individuos en el mundo (el
sujeto que objetiva su entorno termina siendo l mismo vctima de su objetivacin). Se dio
histricamente esta objetivacin en trminos de racionalizacin de la actividad social y
econmica, primero en el mbito del trabajo industrial. El socilogo Andr Gorz19 habla de la
"invencin moderna del trabajo" en estos trminos:
"La organizacin cientfica del trabajo industrial ha sido el esfuerzo constante por separar el trabajo, en
tanto que categora econmica cuantificable, de la persona viviente del trabajador. Ese esfuerzo tom
primero la forma de una mecanizacin no del trabajo sino del propio trabajador: es decir, la forma de la
presin para el rendimiento mediante el ritmo o las cadencias impuestas. () La actividad productiva fue
separada de su sentido, de sus motivaciones y de su objeto para convertirse en el simple medio de ganar
un salario. Dejaba de formar parte de la vida para convertirse en el medio de ganarse la vida. El tiempo
de trabajo y el tiempo de vivir estaban desunidos; el trabajo, sus herramientas, sus productos adquiran
una realidad separada de la del trabajador y dependan de decisiones ajenas. La satisfaccin de
trabajar en comn y el placer de crear eran eliminados en beneficio de las nicas satisfacciones que
puede comprar el dinero. Dicho de otra manera, el trabajo concreto no pudo ser transformado en lo que
Marx llamar el trabajo abstracto ms que haciendo nacer en lugar del obrero-productor el trabajadorconsumidor: es decir, el individuo social que no produce nada de lo que l consume y no consume nada
de lo que l produce; para quien el fin esencial del trabajo es ganar con qu comprar unas mercancas
producidas y definidas por la maquinaria social en su conjunto."

La Modernidad signific entonces, a partir del siglo XVIII, el doble advenimiento:

de una poca en la cual la actividad productiva se separa irremediablemente de su


sentido social, religioso y personal

de una nueva pobreza la del asalariado dos veces despojado: una primera vez de su
cultura campesina de origen (xodo rural y concentracin urbana) y una segunda
vez de una retribucin justa por la venta de su fuerza de trabajo20.

19

Ver: A. Gorz: Metamorfosis del Trabajo, bsqueda del sentido, Ed. Sistema, Barcelona, 1995. Vamos a seguir la sntesis de este
autor para describir los problemas del Mal desarrollo moderno.
20
En los primeros tiempos de la Revolucin industrial, la nica manera para que los nuevos obreros no abandonen su trabajo fue de
pagarles lo mnimo posible para asegurar que necesiten trabajar toda la semana.
21

Portal Educativo de las Amricas


http://www.educoea.org

Cmo ensear tica, Capital Social y Desarrollo en la Universidad?


Mdulo 4: tica y Capital Social: contenidos temticos

Esta situacin, iniciada en Europa occidental, sigue repitindose hoy en numerosos pases
llamados del Tercer Mundo. La trampa del Mal Desarrollo moderno, tal como se practica en
general, reside en el hecho de que la integracin en la dinmica de la produccin
econmica moderna se hace a travs de una dependencia cada vez mayor frente al
dinero y las exigencias y fluctuaciones del mercado laboral. Se vuelve imprescindible tener
el poder adquisitivo para comprar las mercancas y los servicios que ya no son gratuitamente
autoproducidos por la comunidad tradicional desde su red de cooperacin (Capital Social), sino
ms bien producidos por sistemas expertos para ciudadanos clientes, consumidores y usuarios
aislados. Es el lado oscuro del progreso social. La promesa de autonoma se ve entonces
traicionada por una heteronoma real.
"Yo llamo esfera de la heteronoma al conjunto de actividades especializadas que los individuos
tienen que llevar a cabo como funciones coordinadas desde el exterior por una organizacin
preestablecida. En el seno de esta esfera de la heteronoma, la naturaleza y el contenido de las
tareas, as como sus relaciones estn heterodeterminadas de manera que hagan funcionar a los
individuos y a unos colectivos por s mismos complejos como engranajes de una gran mquina
(industrial, burocrtica, militar) o, lo que viene a ser lo mismo, que les hagan realizar a espaldas
los unos de los otros las tareas especializadas que exige una mquina que, en razn de sus
dimensiones y del nmero de sirvientes requeridos, quita a su personal toda posibilidad de
ponerse de acuerdo sobre sus actividades mediante procedimientos de cooperacin
autorregulados (mediante la autogestin). Este es el caso, por ejemplo, de las redes postal,
ferroviaria, area, elctrica, pero tambin de toda industria que recurra a mltiples
establecimientos especializados, a menudo muy alejados los unos de los otros, para proveerse
de los componentes de un mismo producto final. El tipo de colaboracin e integracin en la
esfera de la heteronoma difiere radicalmente de la cooperacin e integracin de los miembros
de un grupo o de una comunidad de trabajo. 21"
El nudo existencial de la problemtica de la modernizacin social reside en el hecho de que esta
nueva cultura no es en realidad la asimilacin de nuevos sentidos y costumbres que podran
valer para los individuos, sino slo la sumisin a los cdigos burocrticos que imponen una
conducta funcional para lograr fines que no son a priori perseguidos voluntariamente por los
agentes. Es decir, que esta pseudo cultura racional de la sociedad burocratizada no es
realmente una cultura en cuanto tal, puesto que no tiene relacin directa con la voluntad y los
valores del individuo, ni le permite comprender la coherencia y los fines globales del sistema en
el cual se siente ms arrastrado que partcipe. El estudiante tenido que "elegir una profesin"
dentro de las carreras universitarias propuestas no escapa a este sometimiento a una conducta
funcional.
Desde luego, el gran reto de la nueva conduccin de la poltica de Desarrollo, desde el
enfoque de tica, Capital Social y Desarrollo, es el de imprimir al conjunto de la sociedad
una dinmica de cambio en la cual los ciudadanos puedan reconocerse como actores
partcipes, puedan entender claramente la finalidad del movimiento y valorarla desde su
propia cultura como algo deseable. Se trata, en otros trminos, de modernizar la modernidad,
realizando efectivamente su proyecto de sociedad de ciudadanos libres y sujetos de su propia
vida. Mientras no se logre esta dinmica que hemos denominada Autodesarrollo, seguiremos en
el crculo vicioso del Mal Desarrollo.

21

A. Gorz, op.cit. p 51.


22

Portal Educativo de las Amricas


http://www.educoea.org

Cmo ensear tica, Capital Social y Desarrollo en la Universidad?


Mdulo 4: tica y Capital Social: contenidos temticos

8.4. Crisis de Capital Social: la socializacin asocial del consumismo


La contradiccin de la modernidad y de su desarrollo aparece ahora claramente: Se sabe que
es necesario, en el marco global de la poltica social, cuidar y promover el Capital Social de la
comunidad, los lazos de confianza, las redes naturales de colaboracin y participacin
ciudadana, pero la evolucin del proceso de modernizacin social como administracin racional
de la "megamaquinaria" (como dice Gorz) promueve naturalmente la fragmentacin y la
desintegracin social de las redes de cooperacin voluntaria en individuos atomizados y
consumidores clientes de bienes y servicios sociales brindados desde el Mercado y el Estado.
Poderosos agentes estructurales de nuestra vida cotidiana fragilizan los lazos sociales y la
posibilidad de darle sentido al voluntariado y la gratuidad:

La monetizacin creciente de la vida cotidiana: La lgica de la compensacin


consumista por los esfuerzos producidos al trabajar se une a la lgica econmica del
aumento constante de la produccin y la apertura de nuevos mercados, y juntas
desarrollan un poderoso proceso de monetizacin creciente de la vida cotidiana, que
incentiva todava ms la actividad de los sistemas expertos y las ciencias. Es decir que
cada vez ms esferas de la vida cotidiana se ven invadidas por relaciones mercantiles:
lo que los individuos hacan gratuitamente por su cuenta antes, hoy se tiene que
comprar pagando los servicios de profesionales especializados.
El consumismo como proceso de socializacin muy peligroso, puesto que se presenta
como una actividad privada, un tipo de "nicho de felicidad privada que permite zafarse
de la suerte comn" (Gorz). Por lo menos es la manera en que la publicidad comercial
nos lo presenta: un placer personal de privilegiado feliz. Esta socializacin
particularmente asocial (la unin con el mundo basada en el placer inmediato del
privilegiado que se distingue del resto por sus elecciones de consumo) nos recuerda el
peligro de anomia denunciado por Durkheim, y afecta sin duda el universo psicoafectivo
del nio pequeo mirando el televisor durante varias horas al da. Dice Gorz: "los
consumos compensatorios son propuestas al individuo privado en tanto que proteccin y
refugio contra el universo colectivo. Constituyen una incitacin a retirarse en la esfera
privada, a privilegiar la bsqueda de beneficios personales y contribuyen as a
desagregar las redes de solidaridad y de ayuda mutua, la cohesin social y familiar, el
sentimiento de pertenencia. El individuo socializado por el consumo no es ya un
individuo socialmente integrado sino un individuo incitado a querer ser l mismo
distinguindose de los otros." (p 68, idem)
Esto determina una mutacin cultural que le quita sentido a la "gratuidad". Primero, la
publicidad comercial mantiene la conviccin de que todo lo que puede un individuo, el
dinero lo puede mejor que l y que los bienes y servicios proporcionados por unos
profesionales remunerados son, por definicin, superiores a lo que uno puede hacer por
s mismo. Segundo, las necesidades de dinero aumentan en la medida en que aumente
la riqueza social, incitando a capas de poblacin hasta entonces no asalariadas a buscar
un trabajo asalariado que, a su vez, acrecienta tambin las necesidades de consumos.
Tercero, "el dinero ganado permite una forma de satisfaccin ms importante que la
prdida de libertad que implica el trabajo funcional. El salario se convierte en el fin
esencial de la actividad hasta tal punto que deja de ser aceptable toda actividad que no
reciba una compensacin monetaria. El dinero suplanta a los otros valores para
transformarse en su nica medida" (idem p 69). Intil de precisar que el trabajo
"voluntario" pierde todo sentido desde este enfoque consumista.

23

Portal Educativo de las Amricas


http://www.educoea.org

Cmo ensear tica, Capital Social y Desarrollo en la Universidad?


Mdulo 4: tica y Capital Social: contenidos temticos

Esta socializacin "asocial" del consumismo termina por disgregar la misma


representacin de la sociedad y del bien comn, y el xito de las tesis neoliberales en
las ltimas dcadas no es sino el signo de cun capaz es el imaginario social
consumista de seducir a los electores y sus gobernantes. Sin embargo, que la ficcin
liberal de una sociedad regulada por el mercado no tenga ningn sustento, un clebre
supuesto imaginario de la teora de los juegos propuesto por Garrett Hardin puede
demostrarlo fcilmente a los estudiantes:
La tragedia de las tierras comunales22.
"En ese supuesto, cada campesino es libre de hacer pastar, conforme a su
inters, un nmero tan alto como sea posible de animales en las tierras
comunales. Cuando los pastos comienzan a estar cargados en su capacidad
mxima, cada vaca adicional que se introduzca en ellos har disminuir el
rendimiento lechero por cabeza.
Pero esta disminucin se har a expensas de todos, mientras que cada
campesino acrecentar su propia produccin lechera aumentando el nmero de
sus animales. Tendr inters en aumentarlo lo ms rpidamente posible, ms
rpidamente que todos los dems. La bsqueda por cada uno de su beneficio
individual terminar, pues, inexorablemente, en la ruina de todos. Unicamente la
limitacin imperativa del nmero total de animales y, en la prctica, de los que
tienen derecho a utilizar esos pastos puede impedir ese desenlace."
Dicho de otro modo, es necesario pensar, para que la sociedad humana pueda existir
un poder de control general orientado al bien comn y el inters de todos, con criterios
ticos y de desarrollo. Como lo dice Michel Crozier23: "Necesitamos ms intervencin
pblica para dominar la complejidad. De una manera u otra todos los grupos - an los
de hombres de negocios - exigen la intervencin". Una cita til, que podr servir de
punto de partida al profesor para tratar este tema, es esta reflexin de Lamennais:
"Entre el fuerte y el dbil
Es la libertad que esclaviza
Y es la ley que libera"

Al terminar este rpido examen del concepto del Mal desarrollo, la modernizacin del mundo se
presenta como un proceso muy ambiguo que se acompaa necesariamente de la destruccin
de las culturas y relaciones sociales tradicionales, y que afecta el Capital Social de la
comunidad a travs de procesos de socializacin asociales. Ser interesante evaluar con los
estudiantes cun poderosos son estos efectos antisociales y cun poderoso es, en cambio,
nuestra capacidad voluntaria de valorar otros sentidos en la existencia, basados en criterios
ticos y solidarios. Tambin ser til debatir en qu medida la heteroprogramacin de la vida
social por los sistemas expertos es ahora una necesidad debido a la complejidad de cada parte
y del conjunto de la actividad tecnocientfica que nos permite vivir, alimentarnos, educarnos,
tener seguridad, curarnos, etc. Una buena pregunta es la de saber en qu aspectos nos es
importante recuperar un control autnomo de nuestra vida, y en qu aspectos podemos
abandonar ese control a los sistemas expertos. Pero, en todo caso, es fundamental no tratar de
disminuir los desafos que implica un enfoque de tica, Capital Social y Desarrollo frente al
22

Garrett Hardin, The Tragedy of the Commons , Science, 162, 1968. Citado en Gorz, op.cit. p 69.
Michel Crozier. "La transicin del paradigma burocrtico a una cultura de gestin pblica".
Reforma y Democracia. Revista del CLAD, Caracas, enero 1996. (citado en Kliksberg, Repensando el
Estado para el desarrollo social, op.cit.)
23

24

Portal Educativo de las Amricas


http://www.educoea.org

Cmo ensear tica, Capital Social y Desarrollo en la Universidad?


Mdulo 4: tica y Capital Social: contenidos temticos

proceso de modernizacin y globalizacin actual que, en muchos aspectos avanza a


contracorriente de la promocin del Capital Social y la autonoma personal y colectiva. Dicho de
otro modo, nada se har espontneamente sin nuestra decisin tica y poltica y sin nuestra
"militancia" voluntaria. Tampoco nada se har fcilmente con ellas.

Los retos del desarrollo desde el enfoque tica, Capital Social y


Desarrollo
En vista de un Desarrollo social tico, debemos asumir el reto de suscitar procesos de cambio y
modernizacin que no afecten el Capital Social, que no vayan a socavar la identidad, unidad,
creatividad y el potencial de autodeterminacin de las poblaciones a las que se quiere ayudar,
sino que, por el contrario contribuyan a su fortalecimiento.
Inventar una estrategia de modernizacin sin etnocidio y con creacin de Capital Social
es el gran desafo de una poltica de desarrollo tica, solidaria y responsable.
La alternativa que hemos planteado desde un enfoque tico es la formulacin de una poltica de
desarrollo en trminos de Autodesarrollo, teniendo la autonoma personal y colectiva como fin
y la participacin ciudadana como medio. Si el proceso de modernizacin del mundo ha
desembocado hasta ahora en un Mal desarrollo que crea cada vez ms heteronoma, significa
que la promesa de autodeterminacin libre del Sujeto moderno no se ha cumplido hasta el
momento, tanto a nivel existencial personal como colectivo. Por lo tanto, necesitamos
modernizar la modernizacin para cumplir con la promesa de una sociedad realmente
autnoma; es decir, una sociedad de Sujetos libres controlando su propio futuro mediante su
accin ciudadana en democracia.
Para garantizar la autonoma de los seres humanos en el proceso de modernizacin, debemos
asegurarnos de que sean Sujetos de su propio cambio, lo que significa:

Ser autor del proceso de modernizacin, y no slo sbdito, o peor an objeto (vctima)
de la modernizacin.

Ser responsable del proceso de modernizacin, es decir responder por l, por las
decisiones tomadas y las consecuencias implicadas.

Esto implica ser capaz de controlar el proceso de modernizacin; es decir, por una parte,
la capacidad terica de conocerlo, analizarlo, reflexionarlo, preverlo y criticarlo; y por otra
parte, la capacidad prctica de tomar decisiones frente al proceso, escoger y rechazar, la
capacidad de decir no a ciertos futuros posibles, la capacidad de dominar el proceso y no
ser dominado por l.

Bajo estas condiciones de autora, responsabilidad, y control, los seres humanos modernos
pueden pensarse como reales Sujetos de su Modernidad, desde luego ser autnomos frente a
ella.

25

Portal Educativo de las Amricas


http://www.educoea.org

Cmo ensear tica, Capital Social y Desarrollo en la Universidad?


Mdulo 4: tica y Capital Social: contenidos temticos

9.1. Qu significa Autodesarrollo?


Inventar otro Desarrollo que sea Autodesarrollo significa dirigir los esfuerzos hacia la
recuperacin de poder, es decir autora, responsabilidad y control por parte de todos los actores
sociales (personas, empresas, sociedad civil organizada, Estados, organismos internacionales),
para el manejo de su Desarrollo. Podemos indicar una lista, no exhaustiva por cierto, de los
significados esenciales de la nocin de Autodesarrollo:
1. La Democracia es la nica forma posible para un Autodesarrollo, que implica que las
personas puedan reunirse, estudiar, criticar, decidir, evaluar y fiscalizar la accin social
comn en libertad. Debe ser concebida de modo articulado a nivel de todos los actores del
Desarrollo: la Persona, la Comunidad local, el Estado poltico y la Comunidad mundial.
Debe tambin comprender no slo la participacin poltica para el nombramiento de
representantes o la elaboracin de leyes, sino tambin la participacin ciudadana en la
definicin de las estrategias de Desarrollo econmico, social y cultural. El paradigma
moderno de la democracia representativa, concebida a nivel del Estado-nacin mediante el
sufragio universal, no es ms que un modelo muy limitado (y muchas veces limitante) de lo
que puede ser la accin democrtica. En Atenas, por ejemplo, las elecciones mediante voto
estaban consideradas como un medio no democrtico sino aristocrtico de elegir a los
mejores (aristoi). La democracia, al contrario, implicaba que todos eran iguales; y por eso
se sorteaban a los responsables polticos entre todos los ciudadanos.
2. El modelo de Desarrollo conducente a la autonoma debe ser formulado en trminos de
bsqueda de equilibrio en lugar de Crecimiento:
Equilibrio personal entre lo que se puede hacer por s mismo y lo que se debe
abandonar a un especialista en el mercado de los servicios y bienes (equilibrio entre la
esfera econmica personal y familiar, y la esfera de autonoma: autocuidado,
autoaprendizaje y autodeterminacin) y no carrera hacia la elevacin permanente de la
renta para pagar servicios cada vez ms caros en un mercado cada vez ms invasor y
destructor de tiempo e independencia. La disminucin de las desigualdades est
ntimamente ligada a la posibilidad de limitar la importancia de la renta familiar bsica para
la satisfaccin de las necesidades de la persona humana.
Equilibrio local entre, por un lado, las necesidades humanas, los recursos disponibles,
los hbitos culturales, los objetivos colectivos consensuados entre ciudadanos e
instituciones locales (municipios, asociaciones, empresas, gobierno regional, etc.) para la
creacin y el fortalecimiento de una red integrada de Desarrollo endgeno, y por el otro
lado, la participacin abierta en el mercado global; y no el sometimiento descontrolado a
demandas industriales exgenas destructoras del tejido social, cultural y ecolgico de una
zona, en nombre del crecimiento del PNB nacional. No habr Desarrollo sostenible global
sin Desarrollo sostenible local autocentrado en el capital social, la cultura y el uso racional
de los ecosistemas, con la finalidad de disminuir las dependencias y, en la medida de lo
posible, lograr la autosuficiencia energtica, econmica, laboral, educativa, y sanitaria. La
proximidad geogrfica y cultural es un poderoso factor de transformacin de las
oportunidades virtuales de libertad en libertad real y calidad de vida.
Equilibrio nacional entre satisfaccin de las libertades instrumentales fundamentales,
segn la expresin de A. Sen, que son: las libertades polticas, las oportunidades
econmicas, los servicios sociales, las garantas de transparencia y la seguridad

26

Portal Educativo de las Amricas


http://www.educoea.org

Cmo ensear tica, Capital Social y Desarrollo en la Universidad?


Mdulo 4: tica y Capital Social: contenidos temticos

protectora24, y participacin en los intercambios internacionales a nivel econmico, poltico y


cultural; y no sometimiento total a la ideologa neoliberal de la globalizacin a ultranza, sin
control ni siquiera de su propia poltica monetaria nacional. Es tiempo de pensar el
Desarrollo de modo sostenible y tico, lo que implica pensar la articulacin entre economa
nacional y economa global25.
Equilibrio internacional entre Naciones soberanas para resolver diplomticamente los
conflictos, pensar la armonizacin de los intereses econmicos, por un lado, y las
exigencias ecolgicas de respeto a los grandes equilibrios de la bioesfera y uso de los
recursos para las generaciones futuras, por otro lado; y no seguir con la dramtica ceguera
actual que hace preferir los pequeos intereses particulares inmediatos a los grandes
intereses universales del futuro. La menor dependencia lograda a nivel local y nacional
debe ayudar los Estados dbiles a recuperar poder de negociacin sobre el manejo de las
instancias internacionales, frente a los Estados hegemnicos.
3. El Control racional del Mercado, para un uso racional del Mercado, es una necesidad
para pensar el Autodesarrollo. Como lo dice Ignacy Sachs, no se trata, a fin de cuentas, del
desarrollo del Mercado sino de la gente desde el Mercado26. Por el contrario, con el auge
de la ideologa neoliberal, el Desarrollo econmico global no tiene control, lo que pone en
grave riesgo el futuro de todas las naciones, los humanos y la bioesfera. Obviamente, slo
otra racionalidad puede limitar del exterior la racionalidad econmica que, de por s, no
tiene modo de autolimitarse, puesto que est basada en la mera maximizacin de los
beneficios y la minimizacin de los costes. Esta otra racionalidad debe ser la razn moral
misma, tal como muchos pensadores de la economa y el Desarrollo lo proclaman cada da
ms27. Para este propsito, es fundamental entender con A. Sen que no es sensato
concebir el crecimiento econmico como un fin en s mismo (op. cit. p. 31). Segn el
mismo autor (a quien no podemos acusar de ser un radical antimercado), existen dos
posturas bsicas acerca del Desarrollo: la de la libertad razonada y la compulsin
econmica (idem. p. 27). La concepcin del Desarrollo como un proceso integrado de
expansin de libertades fundamentales relacionadas entre s (p. 25) permite resistir a la
compulsin que equipara el Desarrollo con el crecimiento del producto nacional bruto, y
recordar que las rentas personales y el PNB no son ms que un medio, y no el nico, para
expandir las libertades (p. 19).
4. El nfasis en el carcter cultural del Desarrollo es una necesidad para pensar el
Autodesarrollo. Como lo hemos visto, nadie puede desarrollar a nadie sin su
consentimiento, y uno no se desarrolla nunca sino a partir de y gracias a lo que es. Apenas
se supera la estrecha visin econmica del Desarrollo, se puede valorar la cultura propia,
ya no como un freno al crecimiento y una limitacin anticuada por superar, sino como el
punto arquimdico de la autonoma y de un proceso de cambio deseado (por ser
endgeno). El auge del enfoque de Capital Social permite ahora esperar un consenso
acerca de la necesidad de apoyarse en la confianza nacional, la transparencia de los
intercambios sociales, la capacidad asociativa y creativa de una comunidad, la calidad de
las redes interpersonales e interinstitucionales, la gobernabilidad democrtica, todas estas
dimensiones del tejido social basadas en la comunidad de interpretacin de la realidad y la
articulacin tradicional de la accin colectiva, es decir la cultura de una comunidad. La
lucha contra el etnocidio y la fragmentacin social se vuelven entonces ejes
24

SEN, A. Desarrollo y libertad, op. cit., p. 17 y p. 57.


Ver: Oswaldo de Rivero: El mito del desarrollo, Fondo de Cultura Econmica, Lima, 2001.
Ver: I. Sachs: La preeminencia de lo social : la reduccin del abismo entre el discurso tico y las duras prcticas del crecimiento
regido por el mercado. http://www.iadb.org/etica/SP4321/DocHit.cfm?DocIndex=71
27
Es todo el sentido de la Iniciativa Interamericana de Capital Social, Etica y Desarrollo promovida por el BID y coordinada por
Bernardo Kliksberg: www.iadb.org/etica
25
26

27

Portal Educativo de las Amricas


http://www.educoea.org

Cmo ensear tica, Capital Social y Desarrollo en la Universidad?


Mdulo 4: tica y Capital Social: contenidos temticos

estratgicos para la eleccin de los programas y mtodos de Desarrollo. Como lo deca


hace ya una dcada el mismo Vicepresidente del Banco Mundial: la claridad de la
identidad cultural y su continuidad evolutiva son esenciales para la creacin de un marco
cultural integrado e integrador, condicin sine qua non de la utilidad y eficacia de
instituciones a la vez enraizadas en la autenticidad y la tradicin y abiertas a la Modernidad
y al cambio. La identidad cultural es esencial para la confianza en s mismo que las
sociedades necesitan para realizar un Desarrollo endgeno. Al respecto, la falta de un
marco cultural duradero tiende a traducirse por una ausencia de confianza nacional y una
fragmentacin social, yuxtaponiendo una elite occidentalizada con una mayora alienada
[]. Esta confianza en s mismo es indispensable si queremos crear un marco cultural que
permita a la modernizacin ser algo ms que un barniz de occidentalizacin28
5. Es imprescindible abandonar una visin universalista y deductiva del Desarrollo como un
modelo one best way aplicable a todos por igual y entenderlo al contrario como la
bsqueda consensual de soluciones creativas a problemas propios de una
comunidad, si queremos darle una oportunidad a la nocin tica del Autodesarrollo. Esto
significa cambiar las rutinas estadsticas de los Expertos demasiado acostumbrados a
transformar la realidad sistemticamente en cifras para despus intentar cambiar estas
cifras, trabajando as con conceptos virtuales alejados de la problemtica concreta. Las
nociones generales abstractas de pobreza, poblacin meta, carencias sociales, etc.,
apenas dejamos el escritorio por salir a la calle o al campo, cobran sentidos muy diversos y
complejos. Por ejemplo, no tiene sentido hablar de extrema pobreza para una poblacin
que vive segn su cultura de modo autrquico, alejada de los centros econmicos, o al
contrario no considerar como pobre una poblacin marginada y discriminada de una gran
metrpolis rica, como es el caso de muchos negros de Nueva York, bajo el pretexto que
sus ingresos superan ciertos lmites calculados por expertos de organismos
internacionales. Cada caso es nico y no se trata de lograr metas estadsticas, sino de
resolver problemas que se presentan y que la comunidad define como suyos y quiere
resolver. Una comunidad selvtica puede considerar como ms urgente solucionar el
problema de la falta de jvenes candidatos a aprender el manejo de los mtodos
medicinales tradicionales, que aquel de la inexistencia de una posta mdica en su aldea.
Autodesarrollo significa tambin devolver a las personas el legtimo derecho de definir
cules son sus propios problemas y cmo deben ser resueltos, primer paso hacia la
participacin, la creatividad, la responsabilidad y el compromiso que garantizarn la eficacia
de las acciones emprendidas.
6. Y, finalmente (last but not least), el modelo del Autodesarrollo es ante todo fundado en una
definicin tica del Desarrollo antes que tcnica, para que la tcnica est al servicio
de la tica. Los ejes del Desarrollo tico pueden resumirse en tres: Solidaridad,
Responsabilidad y Comprensin del otro.
Solidaridad sincrnica con todos los habitantes actuales del planeta, con prioridad para
los que sobreviven en estado de emergencia permanente, lo que significa tratar de modo
desigual a las desigualdades (primero los que no pueden esperar).
Solidaridad diacrnica con las generaciones futuras (Desarrollo sostenible) y tambin
hacia las generaciones pasadas en respeto de su legtimo legado (que sus esfuerzos para
que nosotros podamos vivir, pensar y desarrollarnos no haya sido en vano). Tal solidaridad
diacrnica puede apoyarse en el imperativo categrico de dejar el planeta al salir tal como
nos hubiera gustado encontrarlo al entrar. Vale tambin, para no limitar la perspectiva del
Desarrollo a la inmediatez del presente, recordar el proverbio amerindio que dice que la
28

SERAGELDIN, Ismail. Culture and development in Africa, (1992). Citado en Changement et continuit,
ed. UNESCO, Pars, 1999, p. 7.
28

Portal Educativo de las Amricas


http://www.educoea.org

Cmo ensear tica, Capital Social y Desarrollo en la Universidad?


Mdulo 4: tica y Capital Social: contenidos temticos

Tierra no es un don de nuestros padres, son nuestros hijos que nos la prestan. Nada mejor
para pensar el Desarrollo sostenible que concebirse a s mismo como un mero inquilino
de la Tierra, que tendr que devolverla algn da a sus legtimos propietarios: los hijos por
nacer.
La responsabilidad debe articular la tcnica del Desarrollo con su fin solidario, para que
las soluciones propuestas no participen de la desintegracin del tejido social, el aumento de
las injusticias y el desequilibrio de la bioesfera. La participacin en la toma de decisin
de todos los afectados por la decisin tomada es la clave de tal responsabilidad.
La comprensin del otro es la virtud que permite concebir un dilogo, una participacin,
y una democracia verdaderos. Implica ponerse en el lugar del otro, pero no para ocuparlo
por l (dejndolo sin lugar, aplastado por nuestra propia interpretacin de lo que l es), sino
para dejarlo expresar desde ah su propia originalidad, segn el ejercicio libre de su genuino
modo de expresin. En efecto, un verdadero dilogo interpersonal e intercultural no slo
cuida la libre determinacin de lo dicho, sino tambin de la manera de decirlo.
9.2. La participacin ciudadana y la convivencialidad como valores tcnicos y ticos del
Autodesarrollo
Podemos observar que el enfoque del Autodesarrollo est muy ligado a la percepcin de los
lmites y equilibrio en el uso de las herramientas e instituciones de Desarrollo. Esta visin
sistmica de la problemtica del Desarrollo, abandona para siempre el modelo simplista del
crecimiento econmico ciego como fin, la adquisicin despreocupada de cualquier novedad
moderna como medio, y la reduccin de la gestin de los programas sociales al mero manejo
de estadsticas e indicadores como estrategia. Pero es necesario pensar de qu modo podemos
controlar la eleccin de los fines de los programas sociales y tambin de los medios que nunca
son neutrales. La "participacin" y la "convivencialidad" son conceptos claves para la estrategia
de Autodesarrollo, que significan tambin valores ticos para los agentes de cambio social.
Ocupmonos primero de la participacin:
La participacin
Desde hace varias dcadas, la participacin es un tema de moda en el mundo del Desarrollo
pero es sin duda el mbito en el cual el discurso se separa ms de la prctica. Como lo seala
B. Kliksberg: Los programas sociales hacen mejor uso de los recursos, logran mejor sus metas,
y crean autosostenibilidad si las comunidades pobres a las que se desea favorecer participan
desde el inicio y a lo largo de todo su Desarrollo y comparten la planificacin, la gestin, el
control y la evaluacin.29 Pero esta reconocida superioridad gerencial de la participacin
tiene muchas dificultades a imponerse en la prctica de muchos programas y proyectos de
Desarrollo, tal como los expertos los disean.
Uno de los mayores problemas de los proyectos de Desarrollo, que afecta tanto su carcter
tico como su eficacia y sostenibilidad, es la definicin vertical predeterminada, por parte de
los agentes del Desarrollo, de las carencias de las poblaciones y las respectivas soluciones
racionales a aportar a sus dificultades (soluciones definidas en ltima instancia por las fuentes
financieras!). Aqu encontramos de nuevo el error tico fundamental que consiste en definir para
el otro lo que debe querer, lo que debe ser lo mejor para tal poblacin.
Apostar por la participacin desde el diseo, es superar esta estrategia deductiva del desarrollo
por una estrategia inductiva.
29

KLIKSBERG, B. Hacia una economa con rostro humano, OPSU, Maracaibo, 2002, p. 138.
29

Portal Educativo de las Amricas


http://www.educoea.org

Cmo ensear tica, Capital Social y Desarrollo en la Universidad?


Mdulo 4: tica y Capital Social: contenidos temticos

Este cambio estratgico es fundamental y determina la diferencia entre un proyecto realmente


participativo y un proyecto que finge ser participativo. En un interesante estudio de la
UNESCO30, se menciona la necesidad de distinguir entre pseudo-participacin,
participacin y asociacin, para que las buenas intenciones expresadas no nos engaen
al momento de actuar.
Muchas veces, la participacin se reduce a un proceso rpido y superficial, en el cual se
establece el contacto slo con los miembros ms accesibles, intelectualmente y materialmente,
de la comunidad. Este tipo de participacin no es ms que una obligacin administrativa, que
los tcnicos suelen considerar como mera prdida de tiempo, y que los famosos consultores
internacionales, que se quedan slo unos cuantos das en el lugar del proyecto, antes de
devolver su informe evaluativo, nunca pueden practicar, por falta de tiempo.
Este tipo de experiencia condujo ciertas organizaciones a rechazar totalmente la idea misma de
participacin. Rechazaron al mismo tiempo la intervencin de instituciones y financieras
exteriores, y negaron la pertinencia de la nocin de Desarrollo y de Proyecto. Para los
incondicionales de la autonoma del trabajo de campo, la nica va que quedaba abierta para
una intervencin exterior pasaba por la inmersin en la vida cotidiana de la poblacin con la
cual se trabaja.
En este sentido, el camino estratgico que recorre un equipo de operadores de campo conduce
casi siempre a las mismas conclusiones31:
a. Se necesita una dcada para que un equipo termine por elaborar una estrategia coherente
y defina un modo de intervencin capaz de suscitar un cambio social, tcnico o cultural, con un
impacto satisfactorio.
b. Todos los equipos pasan poco a poco de una lgica de la participacin a una lgica de la
asociacin. Lo que significa:

30

El cambio slo puede venir de una poblacin que siente la necesidad de cambiar.

Cambio y continuidad, Principios e instrumentos para el enfoque cultural del Desarrollo. Ediciones UNESCO, Pars, 1999.

31

Ver: ENGELHARD, P. Cultura, comunicacin y cambio: de la participacin a la asociacin, de la asociacin a la sociedad poltica,
, ENDA-TM, Dakar 1995.

30

Portal Educativo de las Amricas


http://www.educoea.org

Cmo ensear tica, Capital Social y Desarrollo en la Universidad?


Mdulo 4: tica y Capital Social: contenidos temticos

Esta poblacin tiene problemas por resolver (expresin ms satisfactoria que la de


Desarrollo, que a fin de cuentas no significa nada).

La situacin de "pobreza" viene de procesos micro, meso y macrosociales que provocan


secuencias de problemas. Pero puede encontrarse, a nivel local, un inicio de solucin a
estos problemas.

Slo las poblaciones (grupos, comunidades) pueden formular las preguntas y aportar las
respuestas convenientes para solucionar estos problemas, de los cuales no siempre
tienen conciencia inicialmente.

Un agente exterior puede brindarles un apoyo para resolver estos problemas o


formularlos, pero nunca hacerlo en su lugar.

El agente exterior hubiera quizs imaginado otras soluciones, aveces mejores (aunque
nunca se tiene la certeza de ello), pero la ganancia en autoaprendizaje compensa
generalmente el costo de oportunidad de una solucin menos performante.

Si la comunidad cambia en el transcurso del trabajo para solucionar sus problemas, el


"acompaante" debe, tambin, l mismo autocriticarse y comprometer su propio cambio
en el proceso de interaccin social, del cual l no es ms que uno de los componentes.

La explotacin inteligente de las situaciones de cambio revela nuevos recursos y nuevas


capacidades para la solucin de los problemas.

La confrontacin de los respectivos problemas con grupos de pobladores de otra


comunidad crea fuertes oportunidades de cambio y Desarrollo, por la comprensin de
los problemas "sistmica" que supone y ya no "egocntrica". Por eso es muy provechoso
organizar encuentros entre organizaciones comunitarias similares para una
retroalimentacin eficaz y una toma de conciencia global de los mecanismos de Mal y
Buen desarrollo.

Una verdadera sociedad poltica de ciudadanos activos se organiza cuando cada grupo
empieza a percibir la interdependencia de los problemas, las soluciones y las situaciones
de cambio.

El cuadro siguiente permite apreciar la evolucin de la estrategia de Desarrollo, desde la


pseudo-participacin hacia una verdadera relacin de socios:
Tipo de proyecto

Tipo de participacin

Concebido
y
ejecutado
independientemente de la comunidad por
las organizaciones de Desarrollo. Fines y
medios
decididos
de
antemano.
Pertinencia cultural no analizada.

Participacin real limitada o nula.


Comunidad "invitada" a no oponer
obstculos al proyecto. Durabilidad
mnima.

Concebido por las organizaciones de


Desarrollo a partir de una investigacin
detallada de las necesidades de la
comunidad. Comunidad comprometida en
la ejecucin del proyecto. Objetivos

Participacin relativa y condicional.


Proyecto concebido segn criterios e
intereses externos. Durabilidad siempre
limitada por falta de responsabilidad
verdadera.

31

Portal Educativo de las Amricas


http://www.educoea.org

Cmo ensear tica, Capital Social y Desarrollo en la Universidad?


Mdulo 4: tica y Capital Social: contenidos temticos

fijados de antemano, medios ms o


menos flexibles.
El proyecto emerge de la consulta y del
dilogo con la comunidad. La comunidad
es estimulada para formular sus propias
respuestas a sus problemas. Fines y
medios definidos en colaboracin con la
comunidad y, siempre, pueden ser
revisados.

Participacin plena integrada (tendiente


hacia
la
relacin
de
socios).
Identificacin plena de la comunidad con
el proyecto. Probabilidad elevada de
durabilidad del proyecto.

Iniciativa
tomada
por
la
misma
Relacin de socios.
comunidad. La organizacin apoya.
El compromiso institucional se acaba (fin
La comunidad toma la posta.
del proyecto).
Fuente: Changement et continuit, op.cit. p 142

La exigencia de una real participacin de las poblaciones en su propio proceso de


Autodesarrollo pide un cambio profundo en la formulacin y ejecucin de los proyectos. Slo
una verdadera relacin de socios entre una comunidad que decide resolver sus problemas,
organizaciones que quieren apoyarla, e instituciones financieras que sostienen el proceso,
asegura que la poblacin participe realmente de todas las fases claves del proyecto:
diagnstico, diseo, financiamiento, planificacin, ejecucin, fiscalizacin y evaluacin.
Recordemos que "ayudar" significa apoyar la accin del otro, no actuar en su lugar. Adems, el
proceso de asociacin permite una real transparencia de los programas sociales y una
fiscalizacin interna del manejo estratgico y del uso de los recursos, como los Tringulos de
solidaridad lo atestiguan. Evidentemente, la relacin de socios arriesga ser un proceso mucho
ms lento en un principio que la gerencia clsica de programas sociales por parte de entidades
especializadas; pero tambin, un proceso mucho ms eficaz y sostenible en el tiempo.
Pero si la participacin es el procedimiento tico y tcnico el ms deseable, tampoco garantiza
siempre la mejor eleccin. Es importante relacionarlo con un concepto muy til que es "la
convivencialidad".

La convivencialidad
El concepto fue inventado por Ivan Illich (http://www.ivanillich.org/Principal.htm en los aos 7032,
y puede ser de gran utilidad para concretizar la nocin de autonoma en la problemtica del
desarrollo. El trmino designa una relacin del ser humano con las herramientas que usa en la
vida cotidiana33.
En qu medida el uso de una herramienta nos hace ms libres y autnomos o ms dependientes y
heternomos?
32

ILLICH, Ivan. La Convivencialidad, Ed. Barral: Barcelona, 1974.


Illich entiende por herramienta cualquier artefacto inventado por el hombre para servir algn propsito; es decir, no slo lo que se
llama normalmente herramienta, sino tambin las instituciones, los medios de comunicacin, las leyes, etc. Todos los instrumentos
razonados de la accin humana (p. 41).
33

32

Portal Educativo de las Amricas


http://www.educoea.org

Cmo ensear tica, Capital Social y Desarrollo en la Universidad?


Mdulo 4: tica y Capital Social: contenidos temticos

He aqu la problemtica de la convivencialidad.


Nuestra relacin a cierta herramienta es convivencial si ella cumple con su papel de medio y
extiende efectivamente el poder personal o grupal, permite realizar una multitud de fines; es
decir, ampla nuestro grado de libertad y autonoma. La herramienta convivencial responde a las
siguientes exigencias:
es generadora de eficiencia sin degradar la autonoma personal; no suscita ni esclavos ni amos;
expande el radio de accin personal. El hombre necesita de una herramienta con la cual trabajar, y no de
instrumentos que trabajen en su lugar. Necesita de una tecnologa que saque el mejor partido de la
energa y de la imaginacin personales, no de una tecnologa que le avasalle y le programe (idem p. 26).

Al contrario, una herramienta se vuelve no convivencial si la relacin del usuario con ella se
invierte, poniendo el utilizador al servicio de la herramienta, que se transforma entonces ya no
en medio sino en fin en s misma. A medida que una herramienta se vuelve no convivencial, el
ser humano pierde su autonoma frente a ella: su uso se hace obligatorio y su mantenimiento
obliga a gastar mucho tiempo y recursos que no tienen ms finalidad que perpetuar el dominio
de la herramienta sobre sus usuarios. As, se crea una situacin de monopolio en la cual el ser
humano pierde la capacidad de elegir, pierde poder sobre la herramienta, mientras que el
balance de la utilidad de la herramienta se vuelve cada vez ms negativo frente a todos los
problemas que su uso forzoso genera.
Illich propone la teora del doble umbral de mutacin que alcanzan las instituciones
modernas, lo que hemos llamado nosotros (siguiendo a A. Giddens) los sistemas expertos: En
un primer momento, estas instituciones (como el sistema escolar, la medicina, el sistema de
transporte, la industria energtica, etc.) logran alcanzar un umbral que los vuelve muy
performantes, frente a pocas anteriores; fenmeno conocido como el progreso de las
tecnociencias. Pero luego, su uso hegemnico en la sociedad termina produciendo efectos
secundarios negativos que limitan su utilidad directa: el sistema se transforma en un monopolio
radical. Es decir que de medio til para resolver ciertos problemas, el sistema experto se vuelve
fin en s mismo que empieza a alienar la sociedad, obligando a las personas a su consumo,
obligando a los administradores a su gestin, cada vez ms costosa y burocrtica, por los
mismos progresos que el desarrollo de su racionalidad s permite alcanzar.
Yo entiendo por este trmino [monopolio radical], ms que la dominacin de una marca, la de un tipo de
producto. En este caso un proceso de produccin industrial ejerce un control exclusivo sobre la
satisfaccin de una necesidad apremiante excluyendo en ese sentido todo recurso a las actividades no
industriales. Es as como los transportes pueden ejercer el monopolio de la circulacin. Los automviles
pueden moldear una ciudad a su imagen, eliminando prcticamente el desplazamiento a pie o en
bicicleta, como sucede en Los Angeles. () Que la gente se vea obligada a hacerse transportar y se
vuelva impotente para circular sin motor, eso es monopolio radical. (idem p.75)

33

Portal Educativo de las Amricas


http://www.educoea.org

Cmo ensear tica, Capital Social y Desarrollo en la Universidad?


Mdulo 4: tica y Capital Social: contenidos temticos

Por extensin, Illich define lo que llama una sociedad convivencial:


Llamo sociedad convivencial a aquella en que la herramienta moderna est al servicio de la persona
integrada a la colectividad, y no al servicio de un cuerpo de especialistas. Convivencial es la sociedad en
34
la que el hombre controla la herramienta . (p. 13)
Una sociedad convivencial es una sociedad que ofrece al hombre la posibilidad de ejercer la accin ms
autnoma y ms creativa, con ayuda de las herramientas menos controlables por los otros. La
productividad se conjuga en trminos de tener, la convivencialidad en trminos de ser. En tanto que el
incremento de la instrumentacin, pasados los umbrales crticos, produce siempre ms uniformacin
reglamentada, mayor dependencia, explotacin e impotencia, el respeto a los lmites garantizar un libre
florecimiento de la autonoma y de la creatividad humanas (p. 41).

Como se puede constatar, lo ms interesante de la nocin de convivencialidad, que Illich


termina confundiendo con la libertad individual, realizada dentro del proceso de produccin, en
el seno de una sociedad equipada con herramientas eficaces(p.27), es que permite traducir la
nocin de autonoma 35 en trminos de equilibrio y lmites en el uso de los instrumentos
sociales de produccin y consumo. No existen herramientas malas en s mismas (aunque
Illich reconoce que el telfono o la bicicleta son herramientas muy convivenciales, mientras el
automvil es sumamente peligroso en su capacidad de conducir a un monopolio radical) ni
tampoco se trata de rechazar todo progreso tecnolgico, y ni siquiera toda dependencia (con tal
que sea democrticamente pensada y decidida). Lo que s existe son lmites en el uso y abuso
de una lgica productiva ms all de los cuales la autonoma personal y colectiva ya no son
posibles, se crea un consumo obligatorio y se inventan a diario nuevas carencias e
insatisfacciones.
En conclusin, si queremos plantear una nueva estrategia de desarrollo a la vez global y local
basado en el enfoque de tica, Capital Social y Desarrollo, necesitamos cumplir con la agenda
actual (en particular los problemas de gobernabilidad democrtica, empoderamiento y vigilancia
ciudadana, lucha contra la pobreza a travs del fortalecimiento del Capital Social comunitario,
etc.) poniendo en prctica una verdadera participacin, pero con ciudadanos capaces de
reflexionar acerca de la convivencialidad o no de las soluciones que imaginan a sus problemas.
Esto nos remite de nuevo a la Responsabilidad Social de los universitarios y los profesionales
en general, frente a la urgente necesidad de:

compartir la informacin til con la ciudadana

brindarle el acceso a un debate social de calidad sobre sus propios problemas

cubrir las necesidades bsicas de aprendizaje en el uso de la ciudadana. Para


cerrar el crculo de nuestra reflexin, necesitamos finalmente examinar cul debe ser
la actitud tica especfica de este profesional facilitador de procesos de
desarrollo participativo.

Este ser el tema central de nuestro ltimo Mdulo.

34

Illich entiende por herramienta cualquier artefacto inventado por el hombre para servir algn propsito; es decir, no slo lo que se
llama normalmente herramienta, sino tambin las instituciones, los medios de comunicacin, las leyes, etc. Todos los instrumentos
razonados de la accin humana (p. 41).
35
O bien la nocin de libertad tal como A. Sen la emplea (como capacidad para vivir segn su propia agencia) en Desarrollo y
Libertad, op. cit.
34

Portal Educativo de las Amricas


http://www.educoea.org

Cmo ensear tica, Capital Social y Desarrollo en la Universidad?


Mdulo 4: tica y Capital Social: contenidos temticos

Preguntas para el foro


Le proponemos reflexionar sobre los siguientes interrogantes y compartir algunas de sus
reflexiones con sus compaeros de Foro
1. Cmo podra definirse y verse representado el concepto de Capital Social Individual y
Comunitario dentro de la universidad.
2. En qu medida este ejemplo de estrategia de desarrollo ilustra un modo tico de
concebir el desarrollo?
3. Qu indican acerca del ethos poltico y social de nuestros Estados actuales las
resistencias encontradas por A. Fischel para desarrollar los Tringulos de solidaridad?
4. Cmo se expresa esta acumulacin de capital antisocial en tu propio pas?
5. Piensas que los estudiantes estn conscientes de estos problemas?
6. Despus de haber planteado el tema de Capital Social y desarrollo a los estudiantes,
organizar un debate a partir del proverbio Bant: "La fuerza del cocodrilo es el agua".
7. Le proponemos que evale en qu medida la promesa de la modernidad, que es de
hacer de los seres humanos los sujetos libres y autnomos de su propia historia y
desarrollo, ha sido y es cumplida o no, y cmo podramos actuar colectiva y
responsablemente para que lo sea, desde nuestro enfoque de tica, Capital Social y
Desarrollo.

Actividades obligatorias
1. Colectar datos informativos tiles acerca del desarrollo y formularlos en preguntas a
fin de constituir un juego de "Trivial Pursuit del desarrollo"
Instrucciones:
a) Cada participante del curso deber buscar informaciones relevantes, potencialmente
asombrosas y actualizadas, acerca del desarrollo en Amrica Latina o en el mundo, en los
textos mencionados durante el curso o en otros de su eleccin.
b) Una vez colectada la informacin, el participante deber presentarla bajo la forma de
preguntas cerradas con tres alternativas posibles (ver ejemplo), como en un juego de Trivial
Pursuit, agregando la respuesta correcta y la fuente de informacin.
c) El participante ingresar sus preguntas (tantas como quiera) en el foro "Trivial Pursuit del
desarrollo" de su saln virtual, para compartir sus informaciones con los dems participantes,
creando as un material pedaggico para el dictado de un curso de Etica, Capital Social y
Desarrollo

35

Portal Educativo de las Amricas


http://www.educoea.org

Cmo ensear tica, Capital Social y Desarrollo en la Universidad?


Mdulo 4: tica y Capital Social: contenidos temticos

Ejemplo:
En el artculo de Bernardo Kliksberg: America Latina: Una Region En Riesgo. Pobreza,
Inequidad
E
Institucionalidad
Social
http://www.iadb.org/etica/SP4321/DocHit.cfm?DocIndex=329
se puede leer el dato siguiente:
"Un informe reciente de la Comisin Mundial del Agua (Banco Mundial, 1999) calcul que para
adquirir un metro cbico de agua un habitante de los barrios de Lima tiene que pagar 20 veces
el importe que abona un residente urbano, de los estratos medio o alto, que slo abre la canilla
de su casa."
A partir de este dato, se formular por ejemplo la pregunta siguiente:
Para adquirir un metro cbico de agua, un habitante pobre de un barrio marginal de
Lima (Per) sin infraestructura instalada de agua y desage deber pagar al camin
cisterna:
- 2 veces menos
- 5 veces ms
- 20 veces ms
que lo que paga para la misma cantidad un residente urbano de los estratos altos, que
slo abre la canilla de su casa.
Respuesta: 20 veces ms.
Fuente: Banco Mundial, Conferencia sobre desarrollo en Amrica Latina y el Caribe
Valdivia, Chile,1999.
Citado en: Bernardo Kliksberg: America Latina: Una Region En Riesgo. Pobreza,
Inequidad E Institucionalidad Social
http://www.iadb.org/etica/SP4321/DocHit.cfm?DocIndex=329

2. Contestar las siguientes preguntas como si el participante estuviera dictando una


clase de tica, Capital Social y Desarrollo en una casa de estudios superiores:
a) Un estudiante le pregunta durante la clase: "por qu debera yo ser solidario con los pobres
si nadie lo hace y si es intil porque no van a salir as de la pobreza? Acaso tengo yo que
encargarme de los problemas de los dems? Cada uno debe ver por sus propios problemas,
no cree profesor?" qu le contesta?
b) Un estudiante le pregunta durante la clase: "por qu deberamos criticar la modernidad si
nos ha trado tantas buenas cosas, tantos avances cientficos y tecnolgicos? Lo mejor es de
dejar a los cientficos y las empresas trabajar para crear cada vez ms bienestar y poco a poco
el progreso alcanzar a los ms pobres. Es cuestin de esperar y confiar, no cree profesor?"
Qu le contesta?

36

Portal Educativo de las Amricas


http://www.educoea.org

Cmo ensear tica, Capital Social y Desarrollo en la Universidad?


Mdulo 4: tica y Capital Social: contenidos temticos

c) Un estudiante le pregunta durante la clase: "por qu deberamos nosotros como


profesionales promover la participacin de la poblacin en los proyectos de desarrollo si por
definicin ellos no saben tanto como nosotros que tenemos las respuestas ms eficaces y
eficientes a sus problemas?" qu le contesta?
Instrucciones:
- Construir su respuesta en base a los temas tratados en el Mdulo 3 y su experiencia
profesional y personal, tratando de ser convincente y de motivar a su interlocutor.
- La respuesta a cada pregunta no deber exceder de una pgina y media.
- Las respuestas debern ser mandadas por correo electrnico a su tutor en la fecha indicada.
3. Seguir con la actividad de compartir experiencias universitarias significativas para el
enfoque de tica, Capital Social y Desarrollo y la Responsabilidad Social Universitaria en
"RECURSOS" del curso.

37

Portal Educativo de las Amricas


http://www.educoea.org

You might also like